Jump to content

Wikipedia:Reference desk/Science: Difference between revisions

From Wikipedia, the free encyclopedia
Content deleted Content added
Line 589: Line 589:
:Another good question is what is NASA's track record for programs accomplishing the goals they set out to accomplish before the projects get canceled. [[Special:Contributions/76.27.175.80|76.27.175.80]] ([[User talk:76.27.175.80|talk]]) 00:50, 9 July 2011 (UTC)
:Another good question is what is NASA's track record for programs accomplishing the goals they set out to accomplish before the projects get canceled. [[Special:Contributions/76.27.175.80|76.27.175.80]] ([[User talk:76.27.175.80|talk]]) 00:50, 9 July 2011 (UTC)
::That depends on what you think the goals are, and I suspect the goals might be far more advanced than what most people think they are. [[Special:Contributions/99.24.223.58|99.24.223.58]] ([[User talk:99.24.223.58|talk]]) 01:49, 9 July 2011 (UTC)
::That depends on what you think the goals are, and I suspect the goals might be far more advanced than what most people think they are. [[Special:Contributions/99.24.223.58|99.24.223.58]] ([[User talk:99.24.223.58|talk]]) 01:49, 9 July 2011 (UTC)
:::You mean they don't by habit state clear goals outright with time tables such as "we will put a man on the moon by the end of this decade" (though of course not always as big)?[[Special:Contributions/76.27.175.80|76.27.175.80]] ([[User talk:76.27.175.80|talk]]) 02:08, 9 July 2011 (UTC)


== Does special relativistic length contraction "hide" matter? ==
== Does special relativistic length contraction "hide" matter? ==

Revision as of 02:08, 9 July 2011

Welcome to the science section
of the Wikipedia reference desk.
Select a section:
Want a faster answer?

Main page: Help searching Wikipedia

   

How can I get my question answered?

  • Select the section of the desk that best fits the general topic of your question (see the navigation column to the right).
  • Post your question to only one section, providing a short header that gives the topic of your question.
  • Type '~~~~' (that is, four tilde characters) at the end – this signs and dates your contribution so we know who wrote what and when.
  • Don't post personal contact information – it will be removed. Any answers will be provided here.
  • Please be as specific as possible, and include all relevant context – the usefulness of answers may depend on the context.
  • Note:
    • We don't answer (and may remove) questions that require medical diagnosis or legal advice.
    • We don't answer requests for opinions, predictions or debate.
    • We don't do your homework for you, though we'll help you past the stuck point.
    • We don't conduct original research or provide a free source of ideas, but we'll help you find information you need.



How do I answer a question?

Main page: Wikipedia:Reference desk/Guidelines

  • The best answers address the question directly, and back up facts with wikilinks and links to sources. Do not edit others' comments and do not give any medical or legal advice.
See also:


July 4

Connection vagina - uterus

When a woman is pregnant, is this connection closed somehow? I suppose it has to be, to prevent the amniotic liquid to leek. Wikiweek (talk) 00:16, 4 July 2011 (UTC)[reply]

It is the placenta that keeps the amniotic fluid, and thus the developing fetus, in place; birth is imminent (relatively) when the placenta breaks and the amniotic fluid floods out. There is a good diagram in the placenta article. Bielle (talk) 00:25, 4 July 2011 (UTC)[reply]
See cervical effacement, cervical dilation. Wnt (talk) 01:54, 4 July 2011 (UTC)[reply]
The placenta most decidedly is not functioning to keep the amniotic fluid inside the uterus; that's the function of the amniotic sac, as Richard Avery notes. If the placenta is blocking the cervix, it's a dangerous and abnormal situation (the condition is known as placenta praevia; it can cause excessive bleeding at the time of birth and can result in both maternal and fetal death.) - Nunh-huh 06:35, 4 July 2011 (UTC)[reply]
Also cervical mucus plug. Dragons flight (talk) 06:14, 4 July 2011 (UTC)[reply]
See Amniotic sac which explains that the foetus is contained inside a membraneous sac along with the amniotic fluid. This sac is attached to the wall of the uterus by the placenta. When midwives or obstetricians (or anybody) talks about 'breaking the waters' in the final stages of pregnancy it is the rupture, deliberately or naturally, of this sac which occurs. Richard Avery (talk) 06:23, 4 July 2011 (UTC)[reply]

Big thanks to Ruichard Avery and Nunh-huh! That's the last time I post a "fact" I thought I remembered without checking first. It's a good job some people are paying proper attention. Bielle (talk) 06:43, 4 July 2011 (UTC)[reply]

To be clear, the amniotic sac is the watertight barrier - but so far as I know, once the cervix has ripened sufficiently, its breakage is almost inevitable. Note that the chorion, the outer layer of the amniotic sac, which is separated from the amnion in young embryos, is contiguous with the placenta (a tissue with both maternal and fetal components, including the chorion, formed wherever the chorion touches the implantation site of the placenta). Thus saying that the placenta forms part of the barrier is not far off from the mark, even in non-pathological cases where the placenta doesn't line the inside of the cervix. Wnt (talk) 09:01, 4 July 2011 (UTC)[reply]

:: Uhmm, what happened to my response then? Richard Avery (talk) 17:22, 4 July 2011 (UTC) Thank you whoever.Richard Avery (talk) 07:21, 5 July 2011 (UTC)[reply]

Badger page needs help

They are using badger as a paraphyletic page for anything with 'badger' in it's common name, but deceptively have a taxobox which implies something entirely less pathological in my mind. Anyways it screwed up all my region maps because I wasn't watching for insanity. There is probably a lot of mess in there and should be checked for correctness by an expert.Craig Pemberton (talk) 01:41, 4 July 2011 (UTC)[reply]

Futhermore the taxon Melinae is not listed at ITIS, so maybe there is some controversy over this point. Craig Pemberton (talk) 01:58, 4 July 2011 (UTC)[reply]
Why are you posting this here? Are you encountering some sort of unreasonable resistance when you attempt to fix this yourself? Please addres your concerns on the article page or attempt the edits yourself. μηδείς (talk) 23:21, 8 July 2011 (UTC)[reply]

Death, Undead, Zombie

I read the Problems of Definition of Death and we are now entering era where dead is hard to define. It used to be when heart cease beating, some one are prounounced death, but now by using artificial pacemaker, one can continue living. And then people who is braindead is considered dead, but this also challenged by Dr. Franklin Miller. So are these people considered undead? And also if some people with brain damage goes without care, they are neither alive or dead cause of the brain damage, and he still feels hungry and thirst but unable to cook or get descent food so they just ate anything that he found on his way. Can this people be called zombie? And if the people knowledge of medicine advanced, and many of these undeads were taken care, but if a sudden event like war made them untreatable, will we have a zombie apocalypse? roscoe_x (talk) 03:07, 4 July 2011 (UTC)[reply]

I think you are introducing a lot of speculation and unnecessary assumptions. "undead" is not a scientific term, so science would not recognise anyone as "undead". Also just because someone doesn't tick "all the boxes" we have created to define being either dead nor alive, does not mean they are "neither", it just means that our current definitions perhaps are not perfect, it's not impossible that the closer you examine it the line might be so fuzzy that perfect delineated definitions might not even exist, there might be a range, (death spectrum disorder, lol). But these new findings do not indicate that something NEW is happening to humans, it's purely what we're LEARNING about the process, i.e. if we haven't had a zombie apocalypse yet, we are no more likely to have one now just because we are learning more about consciousness and finding new difficulties in the diagnosis of what it really means to be dead. I think it's purely an artefact of looking at the subject on a finer scale then ever before. Vespine (talk) 03:53, 4 July 2011 (UTC)[reply]
An essay I found when looking for this [1] seems more philosophical than practical. A patient is deemed dead because it is well understood he is doomed, but his heart stops; but the transplant of the heart shows it can be started again. I think common sense tells us that someone who is "on the way out", who has passed the usual natural point of irreversible death, can be thought of as effectively dead, in order to save another. "Common sense" involves complex philosophical conclusions, which sometimes (as in this case) are more sophisticated than the theory of ethics to which people appeal. But it is true that with sufficient analysis, some cold-hearted decisions should be apparent. Wnt (talk) 09:10, 4 July 2011 (UTC)[reply]
In an abbreviated extract from The Mikado comic opera someone who was reportedly executed, but should not have been, comes to life to the consternation of the executioner who is thereby caught in a lie to the Mikado.
Enter Nanki-Poo
Nanki-Poo. The Heir Apparent is not slain.
Ko-Ko. Your Majesty, it's like this: It is true that I stated that I had killed Nanki-Poo. ... It's like this: When your Majesty says, "Let a thing be done," it's as good as done — practically, it is done — because your Majesty's will is law. Your Majesty says, "Kill a gentleman," and a gentleman is told off to be killed. Consequently, that gentleman is as good as dead — practically, he is dead — and if he is dead, why not say so?
Mikado. I see. Nothing could possibly be more satisfactory!
All sing the finale. Curtain.
Cuddlyable3 (talk) 13:36, 7 July 2011 (UTC)[reply]

Dehydration - overnight re-ocurrence

On a few occasions I have got dehydrated on a long hike, which gives me a headache, dry mouth, difficulty concentrating, dark urine, and of course a great thirst. During the evening I drink plenty of fluids and recover - symptoms go away and light urine. The next morning I find all the symptoms have returned, though not as bad!

I have wondered why this happens. My theory is that there is some tissue or organ that can only rehydrate water very slowly and that this is taking water from the rest of my body. Is this right? If so which tissue or organ is it? -- Q Chris (talk) 06:28, 4 July 2011 (UTC)[reply]

Yes, I believe that's it. It's most of the tissue in the body, mainly muscle, I think. I know that during dialysis, they have the opposite problem. That is, they can't take the excess fluid out too quickly, because it takes times to migrate from the tissue into the blood stream, where they can get to it. StuRat (talk) 06:42, 4 July 2011 (UTC)[reply]
The Reference desk isn't a suitable place to ask this question. As it has happened to you more than once I suggest you mention it to your doctor, as he will wont to excluded the early symptoms of something else. Let him then tell you how to rehydrate to avoid a rebound. There is also the potential problem of an increased risk of suddenly become incapacitated with heat stroke if you're getting this bad. --Aspro (talk) 12:02, 4 July 2011 (UTC)[reply]
This was not a request for medical advice. I know the medical advice would be "take plenty of water with you when hiking all day in hot weather". It just happens that a couple of times I have underestimated how much to take, i.e the weather has been hotter than I anticipated and/or the terrain more difficult -- Q Chris (talk) 15:05, 4 July 2011 (UTC)[reply]
I've noticed the same thing many times, having done a lot of hiking in the Arizona desert in hot conditions. I don't know the answer for sure, but my hypothesis is that the cause is the balance between sodium and potassium. These are the two main ions in the body, but when you sweat you lose a lot more sodium than potassium, because potassium is mainly located inside cells while sodium is mainly located outside. You restore sodium by eating salty things; you restore potassium mainly by eating fruits or vegetables (or drinking fruit or vegetable juices). If you sweat a lot, it can take a while to get the ions back into proper balance, and in the meantime, the water inside the body moves back and forth between the intracellular and extracellular spaces to maintain osmotic equilibrium. For what its worth, if you hike a lot in hot weather, taking Gatorade (or diluted Gatorade) instead of water will make it easier to stay in good shape. Looie496 (talk) 18:09, 4 July 2011 (UTC)[reply]
Note that there is also an "insensible loss" of water by breathing, etc., even while asleep, especially in desert conditions. Wnt (talk) 13:29, 6 July 2011 (UTC)[reply]
I just experienced exactly the same. This morning I felt kind of like my eyes was bulging out a bit in my head, and my urine was quite dark yellow, and then I came across this question and it all made sense. Yesterday afternoon I did a 45 min. moderate running exercise in sunny weather. Drank some fresh lemon water enroute - continued to drink a lot of water during the evening - feeling quite well hydrated - ate vegetables and eggs for dinner, a bun with mackerel for lunch. I hadn't been doing exercising for the last 3 months. 85.81.121.107 (talk) 06:52, 7 July 2011 (UTC)[reply]

Reccomendations of non-English sources on Evolution

I'm translating Evolution into Interlingua for ia.wiki, seeing as Evolution is a featured article here and we need more featured articles on our wiki. Due to the nature of the language, one of the criteria for being a featured article on the Interlingua Wikipedia is diverse sources: references from more than one source language, and from more than one nation. It's not make-or-break for becoming a featured article, but it'd make the case stronger.

For the Evolution article, I'd like recommendations as to non-English resources to use as primary references. References in languages I am able to read (Spanish, French) are preferred, but I can muddle through just about any language to find a reference if it's suitable. It's not a make or break when it comes to the article being featured, but it'd make the case much stronger.

Asking here because the Evolution talk page asks that only discussion related to altering the article takes place there. Almafeta (talk) 09:35, 4 July 2011 (UTC)[reply]

While a little piece of my scholar's soul cringes at the thought of adding footnotes to an article for the principle purpose of increasing the number of languages used in its references, I would suggest visiting the versions of the article which exist on other-language Wikipedias. While many of those articles do rely heavily on English-language sources, you should be able to find 'local' references as well. (In your preferred languages, see fr:Évolution (biologie) and es:Evolución biológica.) Those references should (in theory, at least) have undergone some measure of scrutiny by native speakers of those languages. TenOfAllTrades(talk) 14:00, 4 July 2011 (UTC)[reply]
I agree. There are some good overview-type sources in the fr article. Itsmejudith (talk) 21:03, 4 July 2011 (UTC)[reply]

Inertial mass

Suppose two positive charges are at rest (initially) close to one another. They'll repel, and begin to speed up. Will their masses increase? By , I would expect that they should, but by E = mc^2 and conservation of energy the masses should be constant. How can the paradox be resolved? 74.15.136.219 (talk) 12:59, 4 July 2011 (UTC)[reply]

There is no paradox. The particles on their own do not form a closed system - you also have to take into account the potential energy stored in the electromagnetic field. It is this potential energy that is transformed into kinetic energy and (if the particles are travelling relativistically) increased mass. Gandalf61 (talk) 13:09, 4 July 2011 (UTC)[reply]
But if we viewed the situation in another reference frame moving relative to the first, wouldn't the momentum of the system appear to increase if the masses of the charges were to increase? 74.15.136.219 (talk) 18:00, 4 July 2011 (UTC)[reply]
The electromagnetic field, having energy, will also have momentum that must be included in the calculation of the total momentum from the point of view of that reference frame. Dauto (talk) 18:20, 4 July 2011 (UTC)[reply]
It is interesting to consider what happens if one charge is an electron and the other an antiproton; the electron should move roughly 1800 times faster and have 1800 times more of the final kinetic energy/relativistic mass. But it isn't actually surprising that the faster moving charge interacts and collects more of the mass-energy from an electromagnetic field. Though it still confuses me a bit to think of "where" the mass in that field originally resides, and what sort of particle it might be described as. Wnt (talk) 18:23, 4 July 2011 (UTC)[reply]
The particles are virtual photons. Dauto (talk) 20:29, 4 July 2011 (UTC)[reply]
Virtual photons with real mass...? Wnt (talk) 19:37, 5 July 2011 (UTC)[reply]
Yes. See the third paragraph of virtual particle. Red Act (talk) 19:50, 5 July 2011 (UTC)[reply]
I hadn't known what to make of that before. It says virtual photons can have mass; but here we see that they do have mass, and a specific mass. The potential energy stored in the field between two charges drops in proportion to the distance between them - because, I take it, the virtual particles going between them have some fixed mass-energy * lifespan that is a "fraction of Planck's constant". The longer the distance, the longer the time, the less mass is held within the field. And the mass is either positive or negative depending on whether the charges are same or different. Interesting. But what fraction of Planck's constant, and is it the same for all virtual photons mediating EM fields? Wnt (talk) 23:58, 5 July 2011 (UTC)[reply]
(I tried to work out this fraction for two electron charges assuming one virtual carrier, but came up with the peculiar result of 0.00116140962 s^2 = 8.9875517873681764E9 / (6.2415096516E18^2 * 6.6260695729E-34 * 299792458). I was thinking energy = FRACTION * Planck's / (X/c) = Coulomb's * charge * charge / distance. Sigh... Wnt (talk) 01:18, 6 July 2011 (UTC)[reply]

Okay great, thanks a lot, I think I understand. Sort of related question: rest mass is invariant, which means it is the same in all reference frames. But this doesn't seem to rule out the possibility that rest mass changes. Is there ever a situation where the rest mass of a fundamental particle changes? 74.15.136.219 (talk) 21:43, 4 July 2011 (UTC)[reply]

No, a fundamental particle of any type all by itself always has the same rest mass. (Some fundamental particles never appear by themselves; see color confinement.) However, a collection of multiple fundamental particles in close proximity won't in general have the same mass as the sum of the masses of the constituent fundamental particles; see mass defect. Red Act (talk) 22:53, 4 July 2011 (UTC)[reply]
In the other hand, if the particle is not by itself it might behave as if it had a different rest mass. see Effective mass (solid-state physics). Dauto (talk) 05:57, 5 July 2011 (UTC)[reply]

Okay, but if the potential energy can be defined up to an arbitrary constant, then isn't the mass created by the potential energy ill-defined? 74.15.136.219 (talk) 17:15, 6 July 2011 (UTC)[reply]

What is the term used to describe baskets of stones or large sacks filled with sand that were used as anchors by the ancient Greeks? This is referenced under Anchor and the History of the Anchor, and I am looking for the specific word that describes it. The word begins with the letter "g." Egudmunsen (talk) 14:27, 4 July 2011 (UTC)[reply]

Gabions? Mikenorton (talk) 17:04, 4 July 2011 (UTC)[reply]

Maybe this should also be posted to the "language" reference desk. Michael Hardy (talk) 20:59, 4 July 2011 (UTC)[reply]

cables question

What is the difference among

-ropes
-cables
-tendons
-wires
-strands
-bars
-rods

in civil engineering(especially in case of prestressed concrete? — Preceding unsigned comment added by 113.199.241.79 (talk) 14:58, 4 July 2011 (UTC)[reply]

There are many differences, but perhaps the most obvious is that some of those are only useful in tension (like cables), while the solid objects (like rods) may also be useful in compression, bending, and torsion. StuRat (talk) 15:06, 4 July 2011 (UTC)[reply]
Making the assumption that this is for commercially available pre-stressed concrete products (pilings, structural members, light posts, etc) you may also find your choice of materials (plain steel, high-strength steel, corten, stainless, aluminum, etc.) will vary greatly. --Romantic Mollusk (talk) 19:46, 4 July 2011 (UTC)[reply]
Complicating StuRat's distinction between "useful only in tension" and "useful in tension and compression" is that sometimes they are distinguished by manufacturing technique instead: Wires are made by drawing through a die; cables are made by bundling a number of wires together. Bars and rods are made through extrusion, and sheets are made through rolling. When I worked in a mechanical testing lab, I occasionally tested wires that would make a decent club, and rods that had to be carefully handled to keep them from bending. --Carnildo (talk) 01:39, 7 July 2011 (UTC)[reply]

Rods and bars are intended to be rigid; ropes, cables, and wires are intended to be flexible. Michael Hardy (talk) 21:02, 4 July 2011 (UTC)[reply]

Opioids and hearing loss

By what mechanism do opioids cause hearing loss? Could one possible cause is excessive ear wax and a lack of ability to blow that ear wax out (and that that is somehow possibly to do with opioids, particularly in individuals that had conditions requiring them to have grommets when younger

So there are two questions here really. Firstly, by what mechanism do opiods generally cause hearing loss? Secondly, is there a plausible alternative mechanism that could be at work?

Re: difficulty in blowing out ear wax. What is really be referred to here is trouble performing the Valsalva maneuver

This is absolutely not a request for medical advice - the background info is provided for precisely that reason - background!

Cheers,

Egg Centric 18:50, 4 July 2011 (UTC)[reply]

When you discuss a medical problem in terms of a specific person and discuss ways of treating it, what you have is a request for medical advice, and asserting that it isn't doesn't change that. Looie496 (talk) 19:34, 4 July 2011 (UTC)[reply]
Perhaps I was unclear but there is absolutely no intent to ask for treatment advice whatsover. I'm interested only in mechanisms of action. Egg Centric 19:40, 4 July 2011 (UTC)[reply]
Nevertheless, I have extensively refactored the question. Hopefully it's clearer now too! Egg Centric 19:46, 4 July 2011 (UTC)[reply]
Hmmm, looking up heroin + deafness on PubMed gets a few papers about heroin causing bilateral sensorineural deafness in really obscure journals that won't even give a clue in the abstract what might be going on. I found another obscure article about oxycodone and hearing loss at PMID 19128688 - a pretty pathetic showing in the scientific literature considering the number of online discussions about people losing their hearing from Oxycontin. There's also a reference to this in the article on Rush Limbaugh. Someone have a PDR handy? Wnt (talk) 19:42, 4 July 2011 (UTC)[reply]

Just as an aside, Valsalva is a method of forcing open a dysfunctional Eustachian tube to aerate the middle ear, and nothing to do with ear wax Si1965 (talk) 20:36, 4 July 2011 (UTC)[reply]

I quickly scanned our ototoxicity article, but saw no mention of opioids. I will have a dig around at work tomorrow Si1965 (talk) 20:43, 4 July 2011 (UTC)[reply]
Ok, well it is theorised that it may feel much like the kind of ear condition that is solved by Valsalva, whatever they may be, as it appears to be found in people who haven't registered with a GP since boarding school or university or some equally distant time who had at least the sensation of blockages there resolved by Valsalva - but same sensation, no resolution after opioids (and definite hearing loss which incidentally is partly resolved by Valsalva) Egg Centric 21:30, 4 July 2011 (UTC)[reply]


July 5

Hello, looking for a book

I search a rich-in-colored-images book about the physiology of the 5 senses. a book that will show almost in every relevant page; rich & informative illustrations about the physiology of those. the reason i ask U guys is because this books are very expensive (more than 120 $), they many times got more then 1000 pages, plus; i have only basic knowledge in the issue and the reason i need such a book is to learn stage by stage, mostly by Re-drawing the informative illustrations in this books.

Thanks, Ben. — Preceding unsigned comment added by 109.67.3.117 (talk) 00:21, 5 July 2011 (UTC)[reply]

Suggestion: 1. Go to amazon.com or other book sources like Worldcat.org. 2. Search for the titles of the expensive books you saw. 3. The page should give you suggestions for similar books, there may be some that are less expensive. --George100 (talk) 10:43, 5 July 2011 (UTC)[reply]
Also note: Worldcat can determine if a nearby library has the book, if you enter your location (although many countries don't have this feature). --George100 (talk) 14:05, 5 July 2011 (UTC)[reply]

Calling all cell molecular biologists and/or biochemists. Article merge needs help

Just a request for an RDS regulars who have a little expertise in cell molcular biology and/or biochemistry and could help out with an article merge. There is a stubby orphaned article at IL-12 and IL-12 Receptor β1 Mutations which would probably do better as a subsection of the main article, which I think is Interleukin 12 but I don't know enough about the topic to actually make the merge properly. See Talk:Interleukin 12 for a little more discussion on the issue. I have also notified Wikipedia:WikiProject Molecular and Cellular Biology, but not knowing how active the project is, I thought I would pitch this out for anyone here to help if they were so inclined. --Jayron32 01:02, 5 July 2011 (UTC)[reply]

Biological trait

Is there a website where I can learn about biological traits? like for example brown-eye brunette man marries a blue-eye blonde woman and they have two boys and two girls. The brunette boy and the brunette had brown eyes like the man and the blonde boy and the blonde girl has blue eyes like the woman. Would it be possible for a blonde boy to have brown eyes and the brunette boy have blue eyes and blonde girl with brown eyes and brunette girl with blue eyes? Also, which eye colour do black hair and red hair usually tend to have the most? — Preceding unsigned comment added by 70.53.229.36 (talk) 01:52, 5 July 2011 (UTC)[reply]

Try Eye color which discusses the genetics and Human hair color which also discusses genetics. --Jayron32 02:01, 5 July 2011 (UTC)[reply]

movie of semen into vagina

I know this may sound porn but I am serious about semen into the woman's vagina in closeup view? what about penis penetrating into the woman's anus in closeup view?—Preceding unsigned comment added by 70.53.229.36 (talk) 01:54, 5 July 2011

Try Google. ~ Mesoderm (talk) 01:56, 5 July 2011 (UTC)[reply]
You are right - it does sound like porn. And you sound like a troll. Go somewhere else. Dolphin (t) 02:03, 5 July 2011 (UTC)[reply]
Such video footage (inside the vagina) has been shown as part of a series on sex on Public Television in the US years ago, likely in the 1990s. I don't recall the name of the series. Edison (talk) 02:12, 5 July 2011 (UTC)[reply]
Video footage of ejaculation from inside the vagina was shown on The Miracle of Life on PBS in the united states, which is the TV show you are likely thinking of (unbelievably, this is a redlink, but the show was fairly notable in its day). It was a 1983 episode of Nova, and won an Emmy according to our article on Nova. The video is widely availible for purchase in many formats at Amazon and other sites. As far as penises entering anuses, such videos are so prevalent on the intrewebz, I don't think the OP needs any help finding thos.e --Jayron32 02:25, 5 July 2011 (UTC)[reply]
Yes, and I think it was black-and-white and looked not much different from a sonogram. Educational but not especially lascivious. National Geographic is more pornographic than that. ←Baseball Bugs What's up, Doc? carrots11:32, 5 July 2011 (UTC)[reply]
a search by with the term 'cream pie' might help --helohe (talk) 10:29, 5 July 2011 (UTC)[reply]
We have an article: Creampie (sexual act). And thank you Jayron for the info. I'd seen that footage years ago but wondered where I'd seen it. Not that I need to know but now at least I can put a name with the memory in my brain. --Dismas|(talk) 10:37, 5 July 2011 (UTC)[reply]
Wikimedia Commons has a number of video resources pertaining to ejaculation, but alas, so far our contributors have not figured out a way to watch semen inside the vagina or uterus with an endoscope. I imagine that the living liquid still knows a few undiscovered tricks not readily apparent outside its favored running track. Wnt (talk) 15:20, 5 July 2011 (UTC)[reply]

Gradient of potential energy

If two point-particles interact with one another, there will be a potential energy function associated with the interaction. Now the (negative of the) gradient of this function is supposed to give the force...but the force on particle 1 or on particle 2? It can't give both, because the gradient of a field is unique, but how can it distinguish between 1 or 2? 74.15.136.219 (talk) 02:13, 5 July 2011 (UTC)[reply]

The force on each particle is identical. See Newton's third law. --Jayron32 02:20, 5 July 2011 (UTC)[reply]
Newton's Third Law of Motion states that if one body exerts a force F on another body, the other body exerts the same force F on the original body. The forces act in opposite direction, but their magnitudes are the same. Dolphin (t) 02:25, 5 July 2011 (UTC)[reply]
Those statements are correct assuming that the only forces arise from the interaction between the two particles. More generally, if you have two particles, the potential energy is a function of two variables (representing the positions of the two particles), and the force on each particle is proportional to the partial derivative of the potential with respect to its position. Looie496 (talk) 02:28, 5 July 2011 (UTC)[reply]

But the force on particle 1 is not the same as the force on particle 2. They're pointing in opposite directions.

I'll give a concrete example. Suppose particle 1 is at the origin, and particle 2 is at a position [x,y,z]. The interaction between the two is the Gravitational force. So, U(r) = -Gm1m2/r. So -∇U = Gm1m2∇(1/r). r = √x^2 + y^2 + z^2, so after some derivatives, I get that -∇U = -Gm1m2/r^3 * r. Now, I know physically that this is the force on particle 2, because the force on particle 1 would be Gm1m2/r^3 * r. But how should I know this mathematically? 74.15.136.219 (talk) 02:42, 5 July 2011 (UTC)[reply]

Because you have to define a direction and keep it consistent throughout your equations. The force on one will be positive and the other will be negative; thus, opposite directions. -RunningOnBrains(talk) 02:48, 5 July 2011 (UTC)[reply]
Where did I define a direction? 74.15.136.219 (talk) 02:52, 5 July 2011 (UTC)[reply]
In your equations, you have written the unit vector "r". This vector holds the direction of the force; in the equations as you have written them, it points towards the center of mass of one of the particles. You may ask, "Which one?" Answer: it doesn't matter! If you choose it to be towards the center of mass of the first particle, then the force on particle 1 will be positive in the "r"-direction and the force on particle 2 will be negative in the "r"-direction. If you choose it to be towards the center of mass of the second particle, then the force on particle 1 will be negative in the "r"-direction and the force on particle 2 will be positive in the "r"-direction. These are physically identical situations, only different in the coordinate system we choose: both give the answer of equal and opposite forces on each particle. -RunningOnBrains(talk) 07:45, 5 July 2011 (UTC)[reply]
The existance of two particles automatically defines a direction. If there are two particles, there is always a line connecting the two particles, and directions can exist relative to this line. In the simplest term, you can consider motion along this line such that the particles move closer together, or conversely farther apart. You can also define directions off the line, for example by defining motion relative to initial starting positions at angles to the line, or you can define an arbitrary coordinate system relative to the line (such that the origin is one of the particles and an axis lies along said line). However, direction automatically exists once you have multiple particles, and motion (and by extension force) just needs to be defined according to arbitrary convention; certain concepts like "towards" and "away from" will be absolutely defined, but other concepts like "negative" or "positive" will need to be defined by the coordinates you choose. --Jayron32 02:57, 5 July 2011 (UTC)[reply]
As Looie said, the potential is a function of the positions of both particles (six real variables), and the force on one is the partial derivative with respect to its coordinates. We may as well choose coordinates such that both particles lie on the x axis, in which case the potential is U(x1, x2) = −K/(x2−x1) where K = Gm1m2 (assuming x2 > x1). Then −∂U/∂x1 = K/(x2−x1)² and −∂U/∂x2 = −K/(x2−x1)², so the forces on particles 1 and 2 are in the +x and −x directions respectively. -- BenRG (talk) 03:16, 5 July 2011 (UTC)[reply]
Ah, great, thanks. Quick question that's not all that related: when is ∂/∂(x1 - x2) = ∂/∂x1? I suspect always, but I can't prove it. 74.15.136.219 (talk) 03:38, 5 July 2011 (UTC) Nevermind, it's obvious now...I need sleep. 74.15.136.219 (talk) 03:56, 5 July 2011 (UTC)[reply]

Hospital double doors

Why do hospital double doors (at least, the ones that I've seen) always open the opposite way from each other (that is, one inward and one outward, instead of both inward or both outward)? —SeekingAnswers (reply) 07:30, 5 July 2011 (UTC)[reply]

To clarify, I'm asking about side-by-side doors, not two doors in succession down a short hallway. —SeekingAnswers (reply) 04:43, 7 July 2011 (UTC)[reply]

so wHeelchair users can open them? (don't seem to have a tilde key on this iPad)
— Preceding unsigned comment added by Robinh (talkcontribs) 03:34, 5 July 2011
The tilde key is two levels down but is on an iPad's keyboard. When you're on the regular keyboard, tap the button for numbers and special characters. Then tap the second button labeled "#+=" for additional special characters. The tilde is in the middle of this keyboard. Dismas|(talk) 12:47, 5 July 2011 (UTC)[reply]
You can also use the "quick sign" link in the Wikipedia edit page; below the text-entry box is an "insert" box with a variety of MediaWiki-syntax hotlinks. Nimur (talk) 14:54, 5 July 2011 (UTC)[reply]
thanks guys. Robinh (talk) 09:49, 7 July 2011 (UTC) and indeed Robinh (talk) 09:49, 7 July 2011 (UTC)[reply]
Hospital doors (and the doors to restaurant kitchens) frequently have hinges that will allow the door to swing either way, and that are hung (or spirally sprung) so the door settles back at the closed position once it's released. This arrangement allows wide objects (or groups) like a cage-cart or a medical team with a gurney to push both doors, allowing quick access. For individual people (who seem to intuitively follow that country's drive-on-the-left/right rule, even when walking), those walking in opposing directions can both push their door, giving the inward/outward pattern you've observed. In a hospital there's an additional benefit to always being able to push a door - you can push a door with your foot or bum or shoulder (to pull you need your hand); that reduces the opportunities for microorganisms to be transferred between people via doors. -- Finlay McWalterTalk 11:03, 5 July 2011 (UTC)[reply]
It's quite common for restaurant kitchens to have 2 doors - one that only opens outwards and one that only opens inwards. That way the waiting staff can hold plates and back through the door, safe in the knowledge that no-one can be coming the other way.--Phil Holmes (talk) 13:14, 5 July 2011 (UTC)[reply]
Thanks for linking 'hospital' and 'door'. I think we're all clear on what we are trying to understand. Caesar's Daddy (talk) 14:06, 5 July 2011 (UTC)[reply]
I'm still unclear on this "open" concept you speak of.  :-) StuRat (talk) 01:33, 6 July 2011 (UTC)[reply]
Haha! DRosenbach (Talk | Contribs) 01:16, 6 July 2011 (UTC)[reply]
My work takes me to a number of hospitals in the UK and I have never encountered doors as you decribe, one leaf opening one way and the other leaf opening in the opposite direction. Sounds bizzare. Do they have instructions or arrows to indicate which way they open, if they do then it is clearly a method of avoiding collisions, if they don't have indicators then, jeez! someone needs to get the door fitters back. Richard Avery (talk) 14:12, 5 July 2011 (UTC)[reply]
You're probably seeing double-egress fire doors. Hospitals are extensively subdivided by fire walls to allow patients to be evacuated from one part into another in case of a fire or smoke condition, rather than out into the street. Doors must be arranged to swing outward in the direction of egress. If there is a choice of directions, as commonly exists at a fire wall (you could go either way), then one door swings one way and the other door the other way. This kind of door is fairly normal in any really large building. Not only am I an architect, but I'm sitting in a hospital right now. I bet there's a set of these doors down the hall. Acroterion (talk) 15:16, 5 July 2011 (UTC)[reply]
As I suspected, they're all over the place here: one almost got me just now when it started to close unexpectedly. Acroterion (talk) 16:25, 5 July 2011 (UTC)[reply]

Raw food

Why is it okay to eat raw fruit and vegetables if it is a bad idea to eat raw meat? —SeekingAnswers (reply) 07:36, 5 July 2011 (UTC)[reply]

Why is it a bad idea to eat raw meat? Do those ideas apply to fruits and vegetables? Given the recent outbreaks of illness claimed to be caused by contaminated fruits or vegetables, are you sure the first part is true? DMacks (talk) 07:41, 5 July 2011 (UTC)[reply]
I'm not sure why you're answering a question with a question but I tend to think that the OP is asking in a general sense. Fruits and vegetables are often eaten raw and the opposite is true of meats. Any e. coli outbreaks that you may be thinking of are more the exception than the rule. Dismas|(talk) 10:40, 5 July 2011 (UTC)[reply]
I was hoping to get the questioner to begin a compare/contrast of the two sets (maybe my response came off as too naive?). Indeed the issues are general, but they really all do come down to "what are the problems with raw meat?" ("pathogens" probably is high on that list) and whether they are a serious concern also for most plant-based foods ("no", with various reasons why). DMacks (talk) 10:48, 5 July 2011 (UTC)[reply]
Listing those "various reasons why" would be helpful. —SeekingAnswers (reply) 16:23, 5 July 2011 (UTC)[reply]
Interesting question. What I have been told before is that things like steak have few bacteria because the insides of say, a cow, do not naturally have a lot of bacteria. It's processed products which take bacteria in the packing environment into the insides of things which are more likely to cause infection. This makes sense because every year it is always sausages and burgers that are mentioned with regards to barbecues and food poisoning. Fresh fruit and veg aren't processed in the same way, and are washed to remove those on the outside. There's also (I believe) a shorter time-to-table for fruit; certainly, fruit people boast about how little time ago your lettuce was still growing in the field. Maybe this gives less time for bacteria to multiply, I'm not sure. Grandiose (me, talk, contribs) 10:47, 5 July 2011 (UTC)[reply]
Also see Steak tartare, Carpaccio, Mett, and Sashimi. And if you want to be particular, prosciutto crudo and beef jerky is also "raw", or at least uncooked. --Stephan Schulz (talk) 11:10, 5 July 2011 (UTC)[reply]
Bacteria, viruses and pathogens that infect plants typically don't infect animals or humans - the biology is just too different. However, despite the vast difference in appearance, cows, pigs, chickens and humans are all very much the same on the cellular level, so bacteria, to a lesser extent viruses, and other pathogens which infect animals can also infect humans. That's why you thoroughly cook chicken, ground beef and pork - to kill off the bacteria and other pathogens which were (potentially) living in/on the animal. While the inside of any healthy animal is usually pathogen free (exceptions for things like trichonosis, etc.) slaughtering an animal is a messy business, and bacteria/pathogens from the skin or the digestive tract can get transferred to the meat. (If you're careful about which animal you butcher and how you do it, you can often get away with raw meat - e.g. tartare, etc.) You don't have to worry so much about carrot pathogens - the main disease concern from vegetables is if pathogen-tainted animal manure gets on it. -- 174.31.196.47 (talk) 15:24, 5 July 2011 (UTC)[reply]
Basically two reasons. First, raw meat is hard to chew and hard to digest -- cooking breaks down the membranes and allows digestive juices better access. Second, the same things that make meat nutritious to people make it nutritious to many microorganisms, which therefore grow much more vigorously on meat than on fruits and vegetables. Looie496 (talk) 16:28, 5 July 2011 (UTC)[reply]
I buy the first reason (although there are other ways to soften up meat besides cooking), but not the second. Fruits and veggies get plenty moldy. It's just that, as noted above, a small amount of fruit mold, unlike a small amount of meat mold, won't hurt you, because it's not the kind of micro-organisms that infect humans. A large amount could harm you, but then you could tell it was rotten and avoid it. StuRat (talk) 01:40, 6 July 2011 (UTC)[reply]
If any part of a fruit or vegetable is moldy you should discard all of it, not just chop the rotten part off, as the natural but harmful carcinogenic mold-chemical will already have spread through most of it. I was trying to find the article written by a scientist in Spiked magazine that describes this, but have not been able to. 92.29.118.212 (talk) 14:37, 6 July 2011 (UTC)[reply]
While I agree with your advice I don't think it's so much that mycotoxins spread themselves. More that the visible portion of the mold is only just the tip of the iceberg. If you find any visible mold colonies, there will usually be a lot of mold that you can't really see. Look at [2] for example. Not what most people would consider fruits or veges but for cereals, seeds and nuts (peanuts and peanut butter are a common concern) aflatoxin is one of the headline mycotoxins. BTW re: another of StuRat's points, while mold (i.e. some types fungi) can be a concern on meat it's more cooked or cured/preserved meats, bacteria are generally the far bigger concern on raw meat. Nil Einne (talk) 19:00, 6 July 2011 (UTC)[reply]
I agree that the mold will have spread much more than just the visible part. In addition, the mycotoxin will have diffused beyond the mold, both visible and invisible. I imagine the mycotoxin prepares the way for the invasion of the mold itself. 2.101.2.194 (talk) 10:46, 7 July 2011 (UTC)[reply]

solar wind temperature

can I ask anyone to say me about temperature of solar wind (energy per particle )please ?[for discussion only not homework ] A. mohammadzade IRAN --78.38.28.3 (talk) 08:32, 5 July 2011 (UTC)[reply]

See Components under Solar wind. Plasmic Physics (talk) 08:45, 5 July 2011 (UTC)[reply]
Also, because the solar wind is a sparse plasma, you may want to read or refresh your memory about a statistical definition of temperature, and also review some of the ways to define the temperature of a plasma. There are many species in the solar wind, and due to the low density, they weakly interact (in most scenarios). So, it's often useful to define a different temperature for, say, the electrons, the hydrogen ions, the helium He+, the helium He2+, and so on. Many researchers therefore call the solar wind a "non-thermal" or "non-thermal-equilibrium" plasma. Nimur (talk) 15:01, 5 July 2011 (UTC)[reply]

why is quantum mechanics weird?

why is quantum mechanics weird? why isn't it more intuitive, like things seem to work in real life (on a non-quantum level). Does this make it less likely to be true? --188.29.236.41 (talk) 10:39, 5 July 2011 (UTC)[reply]

At any scale, things appear to work "differently" than at much closer or much further view. From far away, many details are lost, and if you "only" know the bigger picture, suddenly finding details that don't match what you know is confusing as heck. Quantum is especially weird to many folks because it is so specific and yet so specifically different than some things we have always absolutely known about everything. But it's just a really (really) detailed study that winds up in the same place as the big-picture approximation we usually use.
For example, wave–particle duality seems silly...how can an object behave like a wave, diffract, and be in two places at once? We don't see normal visible objects doing that! Well first, all normal diffraction depends on wavelength and the dimensions of the slit through which it's diffracting. Now if you push the math of the diffraction for large objects, turns out the diffraction wavelength (and therefore the slit dimensions required to diffract it) aren't in the realm of anything we could observe. So the result of the wackiness is still that objects we see only appear to behave as objects (we don't see the wave-like property).
How long did it take before people finally accepted what we currently know to be celestial motion? Sadly it's our understanding and experience that is limited, coupled with assuming everything fits patterns we easily see or expecting that the universe is governed by things that are completely simple. DMacks (talk) 10:58, 5 July 2011 (UTC)[reply]
(ec)It appears to be "weird" because we evolved to interact with what is sometimes called "the Middle World", which exhibits certain properties we get used to and mistakenly assume to hold in general. No, this has negligible influence on the likelihood that it is a useful description of reality. "Truth" is a dangerous word in science - most scientific results are indeed false under a strictly logical interpretation, even though they are "close enough" for any imaginable purpose. --Stephan Schulz (talk) 10:59, 5 July 2011 (UTC)[reply]
(ec back at ya!) Quantum itself is weird, likely weirder than we can possibly understand (can't remember which physicist said something that), but that's because we can't understand it. Even though science often looks for simple explanations and patterns, Nature wasn't designed to be understandable to humans:) DMacks (talk) 11:04, 5 July 2011 (UTC)[reply]
Richard Feynman wrote "I think I can safely say that nobody understands quantum mechanics", and allegedly, "If you think you understand quantum mechanics, you don't understand quantum mechanics". Niels Bohr has said "Anyone who is not shocked by quantum theory has not understood it". --Stephan Schulz (talk) 11:20, 5 July 2011 (UTC)[reply]
"I have no doubt that in reality the future will be vastly more surprising than anything I can imagine. Now my own suspicion is that the Universe is not only queerer than we suppose, but queerer than we can suppose." -- J. B. S. Haldane -- CS Miller (talk) 11:41, 5 July 2011 (UTC)[reply]
I think fundamentally explaining why things appear to behave so differently on really small scales and really big scales is one of the things that a theory of everything would explain. Needless to say, they haven't got there yet. Maybe they will some day, maybe not. Grandiose (me, talk, contribs) 11:13, 5 July 2011 (UTC)[reply]
(ec ec)The world is whooshing through space at high speed, but you can't feel it. Objects thrown fast enough away from it circle it and don't fall down. Apparently solid objects are in fact composed almost entirely of nothingness, with tiny invisible objects pushing and pulling at one another over relatively vast distances to manufacture the illusion of their solidity. Most of your ancestors were yoghurt. The gold in your wedding ring was made in massive exploding star, and 10% of the mass of your body was created by the big bang and has been unchanged since. The solid earth you walk on is but a skin on a boiling pudding of boiling rock. 8% of your DNA is viral rather than human, and almost all your cells need a captive bacterium to live. The world is incredibly older, celestial things are incredibly distant, and observable small objects are so incredibly tiny that they fly in the face of any sensible intuition about things. Your brain evolved to gather fruits and berries and to chase mammoths across the plain, and it's a wonder that it can be adapted so well to understand these ideas, which have very little to do with fruits and mammoths at all, so well. None of these things I've mentioned is intuitive; they're all crazy, really, and we've just gotten used to them. There's no reason to suppose the universe happens to be set up in a manner that we can understand, and it'd be a valid concern to worry that we may reach (or may have reached) points beyond which we're not intellectually able to understand things. So it may very well be that, like cows staring blankly at a passing aeroplane, we're just too stupid to properly understand the quantum world. -- Finlay McWalterTalk 11:20, 5 July 2011 (UTC)[reply]
A religionist might say that quantum mechanics make perfect sense to God. :) ←Baseball Bugs What's up, Doc? carrots11:34, 5 July 2011 (UTC)[reply]

Things in the real world do in fact work because of quantum mechanics, so it isn't that weird at all. A universe that works exactly according to classical mechanics would be much, much weirder than our universe. Count Iblis (talk) 15:30, 5 July 2011 (UTC)[reply]

Take the analogy of someone building things from Leggo bricks and playing with the toys he has created from these. Over time, he gets to know a lot about how such toys work, how they are put together, how much they weigh, how they break up when you drop them. Now, suppose he gets an inside view into just one of those Leggo blocks. Suddenly, he sees it is a completely different world in there, with laws that do not apply to the toys he has created from these blocks. He realises that what he has always seen are is averages, what happens when lots and lost of these elemental blocks are put together. Perhaps, like a lot of sci-fi writers he might have assumed that the very small would just be another version of the very large. But it's not. The world inside a single Leggo block has many features that do not apply to the world we are familiar with. Similarly, the world INSIDE an atom is not like the world around us, which behaves as a gigantic average of countless numbers of these atoms, working in concert. But inside the atom, we see forces and events that have no correspondence to the macro world. That makes sense to me, even if I cannot understand quantum thoery. Earlier in the 19th Century (and even now) some people speculated that there was a world inside the atom which was kind of miniature world of our own. The idea of electrons orbiting a nucleus provided an irresistable analogy with the solar system, and some people took that literally. But it is not the case. The reality is a LOT more interesting than some infinite regression, where the smaller nested versions of the world behave much like the world itself. The "undiscovered countries" of the very small provide us with new laws, and new phenomena which we do not see on macro levels. Myles325a (talk) 05:30, 7 July 2011 (UTC)[reply]

Sun photo

Why pink spots are seen in photographs of sun? — Preceding unsigned comment added by 111Engo (talkcontribs) 12:19, 5 July 2011 (UTC)[reply]

Sundog may be beneficial here. Dismas|(talk) 12:45, 5 July 2011 (UTC)[reply]
It seems that camera gives photo from light in halo inside vacuum space , if sun had light look like leaser beam its photograph was look like any circle--Akbarmohammadzade (talk) 12:54, 5 July 2011 (UTC)[reply]
I'm not exactly sure what you're trying to say, but this has nothing to do with halos, and has nothing to do with light reflecting or diffracting in outer space. This is reflection and/or refraction inside the camera itself; internal parts of the camera that are not specifically meant to do so (mirrors) are very non-reflective (as is necessary for taking high-quality photos), but if a bright enough light such as the sun is shined into the camera, this internal noise can be visible in the finished product.-RunningOnBrains(talk) 13:57, 5 July 2011 (UTC)[reply]
Isn't it just lens flare? CS Miller (talk) 13:00, 5 July 2011 (UTC)[reply]
Yes, it's lens flare. (It's definitely not sun dogs; the would appear white with colored fringes, not red, and they would not be arranged in the pattern seen here. The flares here are also too close to the sun to be dogs.) TenOfAllTrades(talk) 13:10, 5 July 2011 (UTC)[reply]
Indeed, not only do sundogs appear at 22° separation from the sun, but they need to be on clouds, since ice clouds are what does the refracting. Definitely a lens flare or flare-like phenomenon involving reflection on the inner optics of the camera.-RunningOnBrains(talk) 13:53, 5 July 2011 (UTC)[reply]
The clouds don't necessarily have to be conspicuous or even visible (either to the naked eye or in a photograph), however. You can get sun dogs with a very thin haze of ice crystals, if they happen to be in the right part of the sky. TenOfAllTrades(talk) 14:10, 5 July 2011 (UTC)[reply]
Yes, but this looks very much like a six-petal red lens flare, characteristic of a cell-phone camera. Such flares are typically due to internal reflection inside the camera module lens element; and poor filtering of non-visible light wavelengths (due to the use of poor-quality optical glass). Other types of lens flare can be created (and modified) by multielement lenses, or by camera irises, or by lens hoods; but this is pretty characteristic. Even lacking EXIF, an expert could still be able to positively identify the camera that shot this particular example-image. Nimur (talk) 15:16, 5 July 2011 (UTC)[reply]
Not sure of camera issue, but [3][4][5][6] a lot of sun photos show this lens flare, particularly in the case of mid-noon sun. --111Engo (talk) 15:42, 5 July 2011 (UTC)[reply]
It is definitely lens flare, and the colors are caused by chromatic aberration inside the lens.Zzubnik (talk) 08:36, 6 July 2011 (UTC)[reply]
The pink blotches aren't caused by chromatic aberration; CA is a specific image defect caused by slight differences in the refractive index of the lenses at different wavelengths, and would appear as colored 'fringes' around bright points in the image (our article shows several good examples). The blotches are the result of reflections off of the lens surfaces within the camera's optics; the color of the flares is governed by the choice of lens materials and (particularly) the types of antireflection coatings applied to them. TenOfAllTrades(talk) 01:55, 7 July 2011 (UTC)[reply]
My mistake. I presumed abberation is what was causing the light to become colored. Thanks for the clarification. Zzubnik (talk) 08:48, 7 July 2011 (UTC)[reply]

Does the number of radiating spokes correspond to the number of blades of the camera's Iris (diaphragm)? Cuddlyable3 (talk) 09:31, 7 July 2011 (UTC)[reply]

It depends on the root-cause of the lens flare. You may also find our article, bokeh, helpful. Both bokeh and lens-flare are, in practice, highly dependent on the interactions between many different optical interferences: imperfections in the glass; shape of the aperture; shading effects from a lens hood; reflections off the optical system interior surface; and reflections and refractions within the glass and lens elements. Bokeh tends to be a bit more intuitive: a de-focused point of light in the image blurs into a bokeh that is the shape of the iris. Photographers use this for artistic effect. In fact, you can buy "aperture foils" to create kitty-cat-shaped bokehs, pumpkin-shaped bokehs, and so on. In practice, lens flare usually does not have such a straightforward correspondence with the aperture shape, because it usually involves multiple bounces of light. Nimur (talk) 21:12, 8 July 2011 (UTC)[reply]
When the spokes are seen on an on-axis image, what other than a non-circular aperture can determine the angle of the spokes? Cuddlyable3 (talk) 08:26, 9 July 2011 (UTC)[reply]

Watching TV in the dark

Is it bad for your eyes? And what about other screens, like a computer screen or a movie theater screen?Quest09 (talk) 12:27, 5 July 2011 (UTC)[reply]

Why would it be bad for your eyes? Dauto (talk) 15:24, 5 July 2011 (UTC)[reply]
Mom's said that. Could she be wrong? Quest09 (talk) 20:28, 5 July 2011 (UTC)[reply]
Here's a responsible, well-researched article: Television and Health (1961). While the article was published half a century ago, it's thorough, and puts the various health-effects of television in perspective. According to the paper's author, eye-strain is attributable to glare more than any other effect; so, poorly-placed bright lighting is worse than a dark room. Nimur (talk) 15:55, 5 July 2011 (UTC)[reply]
The British and US governments still advise people to avoid glare (light reflected from a screen) [7][8]. The American OSHA also warns against too-bright light, which can cause washed-out and hard-to-read images that lead to eye fatigue.[9] Doesn't seem to be anything about darkness. --Colapeninsula (talk) 12:32, 6 July 2011 (UTC)[reply]
Mothers are never wrong. Your Eye is not designed for constant staring at a small bright image that illuminates only the yellow spot (macula). The Afterimage effect demonstrates the vulnerability of the retina to exhaustion. Therefore take a pause from screen viewing every hour, or more often if you sense tired eyes. Use the pause to scan across a distant view such as the landscape outside. A screen image with adequate brightness and contrast for daytime viewing is probably too bright for use in total darkness. Adjust the contrast and brightness controls for relaxed viewing. Cuddlyable3 (talk) 09:23, 7 July 2011 (UTC)[reply]

Equations

Hello I need help on the following scientific equations as I do not know all the products that is formed.

♦ CO2 + H2O → ?

♦ CO + H2O → ?

♦ SO + H2O → ?

♦ Hydrocarbon + H2O → ?

Please give me the simplest equation. Thank You — Preceding unsigned comment added by 41.14.215.113 (talk) 12:57, 5 July 2011 (UTC)[reply]

This is presumably a homework question. You can find some of the answers at acid rain. Grandiose (me, talk, contribs) 13:15, 5 July 2011 (UTC)[reply]
It's also a duplicate of this question above: Wikipedia:Reference_desk/Science#Nitrogen_oxide.2C_Carbon_dioxide.2C_Sulfur_Oxide.2C_Hydrocarbon; except that this time it uses diamond suit instead of spade. – b_jonas 19:58, 5 July 2011 (UTC)[reply]

I don't know what this is, but might as well say what can be said: CO2= carbon dioxide which forms carbonic acid in water, CO = carbon monoxide, SO = sulfur monoxide, and hydrocarbons can be pretty much anything with CxHx in them. I'm not familiar with any strong reactions with water except for the first, but one journal someone else should readily be able to access gives a Google blurb "Sulfur monoxide reacts with water at OD to form hydrogen sulfide, sulfurous acid, and sulfur".[10] It sounds plausible. Note that carbon monoxide (water gas) can actually be produced from water (in the form of steam) when it passes over carbon, so I really doubt it reacts further with water. Wnt (talk) 21:35, 5 July 2011 (UTC)[reply]

Actually, Carbon monoxide can react with water, in the appropriately named Water gas shift reaction. Buddy431 (talk) 04:28, 6 July 2011 (UTC)[reply]
This does not happen at STP, however:
I think that carbon monoxide should be quite stable in solution, but if it does react the first point of attack should be the carbon atom. This should form a formyl and a hydroxyl radical, this in turn, reacts to form a formic acid molecule. According to classical thinking, carbonous acid is known as carbonous hydrate or hydrate of carbon monoxide, but this not formed as carbonous acid is highly unstable in solution. It is even known that carbonous acid is a protomer of formic acid in the gas phase. Plasmic Physics (talk) 14:00, 25 May 2011 (UTC)

Tree Falling

Hello I would like to have some help on this difficult equation for for my science project. If a tree falls in Knysna woods in South-Africa,Western Cape at the co-ordinates : 34°02`00.00"S 23°04`00,00"E will it make a sound if there is no one to hear it? ♦The tree is a Knysna Yellowwood, ♦The branches is reddish-yellow ♦The tree is falling at 5 metres per second, ♦At a angle of 45° ♦A mildly breeze of 7knots is blowing from the west ♦The tree is falling North-South

Please help me. — Preceding unsigned comment added by 41.14.215.113 (talk) 13:13, 5 July 2011 (UTC)[reply]

If a tree falls in a forest should answer your question (I'm assuming good faith, but it's straining). Grandiose (me, talk, contribs) 13:17, 5 July 2011 (UTC)[reply]
You could design an experiment to investigate it, using a pencil, a mechanism to make the pencil fall when you are outside the room, and a video camera. What do you think the result will be? Will the experiment be valid if you aren't in the room but your video camera is? Itsmejudith (talk) 14:53, 5 July 2011 (UTC)[reply]
I'm disappointed that the article has no QM discussion. The trick with this sentence is the word "hear", as sound carries in many ways - such as alteration of weather by the butterfly effect. A tree on an island will fall or not, but a tree in a true Schroedinger's Cat grade "box" exists in a superposition of states. One comes to that island and determines, on opening the box, that a tree has fallen; hence it must have made a sound, you'd think - but if the sound is such to dissipate in a finite time, there may be no evidence of it when the tree finally is determined to have fallen. Yet when the tree actually fell the sound was cancelled out in the superposition of states by all the other out-of-phase sounds of the tree falling a moment sooner or later. Wnt (talk) 15:11, 5 July 2011 (UTC)[reply]
I disagree with your last point that the superposition means that the sound was cancelled out. The superposition you get here involves the sound being entangled with many other degrees of freedom. It's then not a coherent superposition in which you only have to add up the amplitudes of the sound. Count Iblis (talk) 17:12, 5 July 2011 (UTC)[reply]
To be honest? I still haven't quite figured that out. And I'm still thinking that if you had some observation device to measure pressure at one point within a large number of such tree-boxes as the sound (might be) being made, there would be no overall difference in probability of a high or low result at any point; hence no pressure waves exist... Wnt (talk) 17:56, 7 July 2011 (UTC)[reply]

You neglected to say how big the tree is. If it is a full-size Kynsna Yellowwood (Podocarpus latifolius), that's a huge tree, and somebody would surely hear it fall, since the nearest houses are only 200 meters away, up the hillside. Looie496 (talk) 17:26, 5 July 2011 (UTC)[reply]

The OP asked "will it make a sound". Sound arises from sound waves, right? And those waves will be produced whether someone happens to be around or not. ←Baseball Bugs What's up, Doc? carrots18:08, 5 July 2011 (UTC)[reply]
You are defining "sound" as being the sound waves and not the mental process of turning those sound waves into something we call sound. Once you define what "sound" means, the question is easy to answer. But, you are changing the question by doing so. As it was asked, there was no definition for what "sound" means. -- kainaw 18:21, 5 July 2011 (UTC)[reply]
Then he needs to restate the question more clearly. He also needs to explain what "no one" means. Does it mean no human being? Or no creature of any kind that is capable of hearing? ←Baseball Bugs What's up, Doc? carrots18:26, 5 July 2011 (UTC)[reply]
That is the entire point of the question. It cannot be answered because it is ambiguous. It is the same as "What is the sound of one hand clapping?" You have to define what clapping means. Is it the action of two palms striking one another? Is it the waving motion of an arm? For a scientific person, these are rather inane questions, but they are still very popular. -- kainaw 18:30, 5 July 2011 (UTC)[reply]
Goddess only knows why. "How many angels can dance on the head of a pin?" There are two possible answers: (1) All of them; or (2) None of them. ←Baseball Bugs What's up, Doc? carrots18:34, 5 July 2011 (UTC)[reply]
You forgot one possibility: (3) Some of them.-RunningOnBrains(talk) 19:20, 5 July 2011 (UTC)[reply]
Give the OP points for adding a lot of irrelevant detail to make it look more like a question. Next he'll be showing us the proof that 1 = 2. :) ←Baseball Bugs What's up, Doc? carrots18:36, 5 July 2011 (UTC)[reply]
EO says "to clap" originates from a Frissian (sp?) verb meaning "to beat". Hence the term "clapper" for the clanging thing in a bell - which of course won't clang unless there's something to clang against. ←Baseball Bugs What's up, Doc? carrots18:42, 5 July 2011 (UTC)[reply]
Or the old fallacy, "If an irrestible force is approaching an immovable object, what will happen?" The answer is that nothing will happen, because by definition those two entities cannot exist in the same universe. ←Baseball Bugs What's up, Doc? carrots18:56, 5 July 2011 (UTC)[reply]
Yes they can. I'm an immovable object and I encounter irrestible forces all the time. But irresistible forces, ah, they're another matter. -- Jack of Oz [your turn] 19:27, 5 July 2011 (UTC)[reply]
Do you resist them? Or do they move you? ←Baseball Bugs What's up, Doc? carrots20:05, 5 July 2011 (UTC)[reply]
Iain Banks's answer is "The unstoppable force stops, the immovable object moves". But this depends on the question being formulated as "unstoppable" rather than "irresistable". Tevildo (talk) 22:23, 5 July 2011 (UTC)[reply]
Then the force and the object weren't really unstoppable and immovable. Unless maybe they chose to stop and to move. ←Baseball Bugs What's up, Doc? carrots06:37, 6 July 2011 (UTC)[reply]

(outdent) "The tree is falling at 5 metres per second, At a angle of 45°" This leaves the tree in the peculiar situation of falling at a constant speed and maintaining a constant angle. Was the tree dropped from a crane or a helicopter? Wanderer57 (talk) 05:11, 6 July 2011 (UTC)[reply]

Is the answer that the tree hasn't hit the ground yet? So it hasn't yet made a sound (although a tree does make a lot of creaking and branches hitting other branches before the big crash comes).Itsmejudith (talk) 06:56, 6 July 2011 (UTC)[reply]
That would explain why it's falling north-south rather then simply north or south (which would be the case if it was uprooted or broke) Nil Einne (talk) 07:10, 6 July 2011 (UTC)[reply]

This boils down to the philosophical discussion of "what is sound". Are vibrations in air molecules "sound" before an ear has heard them? — Preceding unsigned comment added by Zzubnik (talkcontribs) 08:48, 6 July 2011 (UTC)[reply]

I class this as a troll question because it is an ornamented crib of the hoary old (1883) koan If a tree falls in a forest (a Wikipedia article) and I contend that it was never an equation for a science project. People are having such fun with it that I won't delete it. I had more fun with the variation "A stealth fighter plane crashes in a forest with no one around, does it make a sound?" to which my answer is "Yes then no.". Cuddlyable3 (talk) 09:04, 7 July 2011 (UTC)[reply]

Electron cyclotron resonance and tapping

An electron in a static and uniform magnetic field will move in a circle due to the Lorentz force then if a electromagnetic wave of the right frequency (the same as the electrons circular orbit frequency) then the electron would gain energy is their anyway for the electrons energy to be directly turned into electricity? — Preceding unsigned comment added by 82.38.96.241 (talk) 15:16, 5 July 2011 (UTC)[reply]

There is no efficient way to extract large quantities of electric current from electron cyclotron motion. Strictly speaking, when you detect cyclotron frequency using a radio-receiver, your antenna is collecting electromagnetic energy and a current is being induced in the antenna; so "electricity is being produced." But the quantity of energy collected this way is tiny, compared to the amount needed to create the plasma in the first place. Nimur (talk) 16:59, 5 July 2011 (UTC)[reply]
See Homopolar generator for a device that uses the Lorentz force to generate electricity, albeit from mechanical rather than electromagnetic energy. Tevildo (talk) 22:18, 5 July 2011 (UTC)[reply]

The Pineal Gland - do we have 2 of those?

it says in the Wiki article; "Its shape resembles a tiny pine cone (hence its name), and it is located near the centre of the brain, between the two hemispheres, tucked in a groove where the two rounded thalamic bodies join."

therefore i ask U guys. thanks. — Preceding unsigned comment added by 109.67.3.117 (talk) 15:46, 5 July 2011 (UTC)[reply]

No, we only have one Pineal gland each. I can't see how you got that we have two from the quote you have given. --TammyMoet (talk) 15:58, 5 July 2011 (UTC)[reply]
Let me add that the fact that we only have one is the reason why Descartes proposed that the pineal gland is the location of the link between the physical brain and the nonphysical mind. Looie496 (talk) 16:18, 5 July 2011 (UTC)[reply]
Maybe "a" reason, but the location had something to do with it too. 99.24.223.58 (talk) 20:37, 5 July 2011 (UTC)[reply]
If you look here and scroll down to the third (slightly blurry) diagram you can see quite easily how the pineal fits in between the two hemispheres. Richard Avery (talk) 22:14, 5 July 2011 (UTC)[reply]
Is it safe to say that anything in the midsagittal plane exists as an unpaired organ? DRosenbach (Talk | Contribs) 01:11, 6 July 2011 (UTC)[reply]

Hello, i'll say why i thought we have 2.

i understand from a neurobiologist we have X2 of any brain-organ; for example; 2 Thalamuses, 2 Hypothalamuses, and so forth.. by that, plus the written in Wiki, i asked if we have 1 or also two of this.

i understand from the first response indeed we have X2 PG. — Preceding unsigned comment added by 109.67.3.117 (talk) 07:00, 6 July 2011 (UTC)[reply]

I suggest you show your neurobiologist acquaintance these reponses and see what he says. Richard Avery (talk) 07:18, 6 July 2011 (UTC)[reply]
As Richard suggests, you are mistaken and have presumably misunderstood the neurobiologist. There is only one thalamus and one hypothalamus in any one brain: however, both these organs have paired lobes - linked halves lying either side of the midline of the brain - just as the brain itself has one cerebrum divided laterally into two linked cerebral hemispheres without being two cerebrums or two brains in one head. {The poster formerly known as 87.81.230.195} 90.201.110.220 (talk) 10:36, 6 July 2011 (UTC)[reply]

Okey, Thanks, i understand what you are saying. but i still must ask what will you say about the following?:

http://en.wikipedia.org/wiki/File:Thalamus.gif

i see here 2 Thalamus's, 1 in every hemisphere. it's clear to me we have to refer it is ONE Thal' spllited into the 2 hemispheres. it's funny though..

thanks anyway. — Preceding unsigned comment added by 79.182.28.217 (talk) 02:36, 7 July 2011 (UTC)[reply]

I see 1 thalamus with 2 lobes joined by a stalk. Great animation. Richard Avery (talk) 06:36, 7 July 2011 (UTC)[reply]

Let me try to state the situation a little more clearly. The brain is essentially symmetric, in that every structure on the left of the midline has a matching structure on the right. In most cases the left and right structures are separated by intervening structures (i.e., they form two distinct parts), but there are a few structures where this does not happen, because they impinge on the midline. These include several nuclei in the thalamus, epithalamus (which contains the pineal gland), hypothalamus, and brainstem. Looie496 (talk) 18:25, 7 July 2011 (UTC)[reply]

Questions about anti-entropic matter

Years ago, I read an article in a science journal (it might have been SciAm) about a new study using computer modelling of anti-entropic matter which found, to everyone's surprise, that anti-entropic matter is more robust than originally thought, and that it resists encounters with entropic matter better than expected. It might therefore be possible that there is still anti-entropic matter in the Universe -- that is, matter which is going from a state of less to more order.

Since then, I have been very curious about the properties of anti-entropic matter. For example, what would an anti-entropic star look like? I can't even imagine how it might appear from the perspective of someone looking at it through a telescope. If sufficient anti-entropic matter collapsed to form a hypermass, how would an anti-entropic black hole behave? I've tried asking these questions in a number of places over the years, but no one has ever given me a satisfactory answer. I even tried some of the science websites run by scientists, and haven't gotten a response.

Anyone care to take a crack at answering? -- SmashTheState (talk) 20:56, 5 July 2011 (UTC)[reply]

Sure. Physicists do not recognize any such thing as anti-entropic matter. It is a conceptual error, usually associated with a belief in teleology. Looie496 (talk) 21:13, 5 July 2011 (UTC)[reply]
I can't find the article I read in a dead-tree journal with Google (I searched SciAm and it wasn't in there, so it was another journal), but I do recall reading about computer simulations of anti-entropic matter, and it mentioned that, as you say, physicists had previously believed that it was not possible for any anti-entropic matter to have survived the early stages of the Universe because it would have been rapidly made entropic by interaction with entropic matter. This computer model, however, showed that anti-entropic matter is much more resistant than previously thought, and that it's possible anti-entropic matter could still therefore exist. As I understand it, there is nothing preventing anti-entropic matter from existing, on a theoretical level. The arrow of entropy is entirely arbitrary, and can point in either direction on a quantum scale. My question, therefore, is not a conceptual error. While an anti-entropic star might not exist, the idea is not internally inconsistent. I'm asking for a theoretical answer of how such a thing would appear through a telescope, were it to exist. SmashTheState (talk) 23:12, 5 July 2011 (UTC)[reply]
Let's put it this way: if there is such a thing, it would really be a sight to behold! Suppose I have a Lotto machine (a big tumbler) full of Lotto balls (numbered ping-pong balls) and the machine and balls are made of this stuff. Then it starts out all random and jumbled, but as I turn the crank, they neatly jump up into the tube the balls are released from? Do they end up all neatly in order, from 1 to 40, all facing the same direction with the numbers right side up? Who says that happens? Now, I'm not a great believer in causality, but mixing plus and minus time really confuses me. And that's not even getting into what happens if you dump in half entropic matter and half anti-entropic matter, or if to conserve resources you spray paint normal Lotto balls with anti-entropic matter. (OTOH it might be lucrative research! ;) ) Wnt (talk) 21:42, 5 July 2011 (UTC)[reply]
Come to think of it, if there was anti-entropic matter dispersed in space, as gas or dust particles, I suppose they would all have coalesced into one giant crystal in one place, somewhere. Or perhaps some other work. Actually, as sci-fi, I kind of like the idea - it seems like a plausible explanation for the First Berserker, set aside and ordained from the beginning of time by divine pity, for the moment when mankind must be saved from ascension. Wnt (talk) 21:49, 5 July 2011 (UTC)[reply]
There are serious arguments about this. You can consider a universe that instead of global low intial entopy condition has different sectors in which the time's arrow as defined by the entropy, runs in the opposite directions. Then you can look at what happens at the boundary of such regions. Another recently proposed idea is that the universe started out from an inflating patch but that patch is just a minimum of the entropy, so time runs in both directions from that patch. This means that that we can exist both now and also 27.4 billion years "ago" :) .Count Iblis (talk) 23:23, 5 July 2011 (UTC)[reply]
I can understand having variable, random entropy so that sometimes the "arrow of time" seems to reverse. Tumble a Lotto machine of ordinary balls long enough, and for one shining moment a snapshot could capture 1 to 40 neatly lined up in order. The part that gives me trouble is that some matter would be intrinsically set to increase in entropy and some to decrease at the same time in the same general region of space. Wnt (talk) 23:42, 5 July 2011 (UTC)[reply]

Crystals accumulate ordered mass, but they are a solid state phenomenon which does not occur in the other states of matter, and they can not form from other solids, only fluids. Life can also accumulate ordered mass in fluids but not ionized plasma. Crystals and life both depend on external sources of energy to continue to grow. 99.24.223.58 (talk) 03:18, 6 July 2011 (UTC)[reply]

I'd just like to note that no one yet has taken a guess at what an anti-entropic star (or rather, a star made of anti-entropic matter, because the two may be different) would look like. If no one here is qualified to answer, I'd be grateful if someone could suggest a forum where physicists hang out that may be able to provide an answer. (Other than Straight Dope -- I'm banned.) -- SmashTheState (talk) 22:07, 6 July 2011 (UTC)[reply]
Neutron stars might become ordered in a way similar to crystals as they cool, but I can't remember. You should check ArXiv. They are a forum. 99.24.223.58 (talk) 19:39, 8 July 2011 (UTC)[reply]
In particular, [11]. 99.24.223.58 (talk) 19:57, 8 July 2011 (UTC)[reply]

mouthwash

Can I make any good cocktails using mouthwash as an ingredient? — Preceding unsigned comment added by 86.128.100.251 (talk) 21:02, 5 July 2011 (UTC)[reply]

Probably not. This SDMB article goes into the unpleasant details. Tevildo (talk) 22:21, 5 July 2011 (UTC)[reply]
There is a reason for the "Do not swallow" on the bottle. It is toxic.--Shantavira|feed me 05:49, 6 July 2011 (UTC)[reply]
A greenish or bluish version of "white lightnin'". ←Baseball Bugs What's up, Doc? carrots10:32, 6 July 2011 (UTC)[reply]
A scotsman told me that Whisky is an excellent mouthwash as long as you do not spit it out. Cuddlyable3 (talk) 08:45, 7 July 2011 (UTC)[reply]

Roundworms in my cat

(I should start by saying that I am taking my cat to the vet, so I'm not asking for medical advice here.) Just today I observed a roundworm in my cat's vomit. It was like a huge, squirmy spaghetti noodle; quite possibly one of the most horrifying things I've ever seen. So, I've done some research about this, but a lot it is just regurgitated (no pun intended) generic pet-blog info. What I really want to know is what to expect going forward. I wonder if anyone has had an experience with this, and, in the course of dealing with it, come across any good educational resources. Basically, I want to educate myself as much as possible BEFORE visiting my vet, so that I know what kinds of questions to ask her. Also, I'd like to make my cat as comfortable as possible while it goes through the de-worming. Any info/insights would be welcomed. Quinn BEAUTIFUL DAY 23:17, 5 July 2011 (UTC)[reply]

"The prognosis of a roundworm infection is good if appropriate medication is given promptly. However, in some instances, extremely debilitated kittens may die.... Roundworms can be a health risk to humans."[12] 99.24.223.58 (talk) 00:31, 6 July 2011 (UTC)[reply]
Since people can get them, too, you might want to restrict the cat to one room, with food, water, and a litter box, to limit the chance of them spreading, until cured. Also be sure to wear rubber gloves when changing the litter box, and don't let you cat out, where it could infect other cats and people. StuRat (talk)
Basically what they will do is give your cat some kind of medicine (I believe roundworms is a "glop of stuff down the throat" sort of thing), and the cat will be fine with it and poop out a bunch of dead worms. In the meantime, keep the cat from spreading it to others by making sure their vomit and poop are isolated and thrown out immediately. It's not likely that you'll contract roundworms yourself unless you are eating things from the garden when the cat does their thing. (You have to eat the eggs. Bleh.) Anyway this is a pretty routine part of cat ownership in my experience. My cat had worms a number of times in its stool, and it was an easy fix each time. It's about the grossest thing I can imagine but as far as cat diseases go, it's very easy to deal with. --Mr.98 (talk) 13:56, 6 July 2011 (UTC)[reply]
The vet will ask you what the worm looked like because there are different pills for round and flat worms. Giving a pill to a cat takes some skill and determination. Cuddlyable3 (talk) 08:42, 7 July 2011 (UTC)[reply]

July 6

Black hole powered slingshots

Suppose there was a 10,000 solar mass black hole without an accretion disk five light years from Earth, orbiting the galactic center along with the rest of the stellar neighborhood with a typical relative motion in a random direction. How much velocity could a robot ship powered with boron-11 fusion and nitrogen propellant achieve with Gravity assist#Powered slingshots? 99.24.223.58 (talk) 00:24, 6 July 2011 (UTC)[reply]

Note that to get the most boost, you need to do multiple slingshots, so it would be useful to have two or more massive objects close to each other and sling back and forth between them. StuRat (talk) 01:45, 6 July 2011 (UTC)[reply]
  • It's not possible to slingshot off of a single object.
  • Taking the galactic center as the second object, the slingshot speed will be of the order of the orbital speed of the black hole.
  • The mass of the black hole is not important.
Dauto (talk) 02:15, 6 July 2011 (UTC)[reply]

[[Akbar Mohammadzade]]: somebody used to say something about black holes , I prefer to say about supernovae , the first one is death and being secretary , the second one shows celebrating and brightness , and life---------------

gravity field of black hole dosenot differ with first star in outer space than event horison .what this says us ?It says that the massive 10000 solar mas star is equal to such black hole from distance which you are monitoring , dont got afraid of black hole , it is not much more scary from far away so distance .we may near by to any black hole 1 AU and our space craft only do rotate 1.4 times more than earth speed roud sun.

--78.38.28.3 (talk) 05:51, 6 July 2011 (UTC)Akbar Mohammadzade>>M2=2M1 &d2=d1 ==> V2=1.4V1 <<  ;;;M2=10000M1 ==>V2=100V1--78.38.28.3 (talk) 13:43, 6 July 2011 (UTC)[reply]

This....really does not make any sense.-RunningOnBrains(talk) 07:04, 6 July 2011 (UTC)[reply]

>>M2=1.4M1 d2=d1 V2=1.2V1 << — Preceding unsigned comment added by 78.38.28.3 (talk) 07:06, 6 July 2011 (UTC)[reply]

Note, from the section title, that the questioner is asking about a powered slingshot exploiting the Oberth effect, not just an unpowered gravitational assist. The answer will therefore depend on the capabilities of the hypothetical spacecraft - specifically, its delta-v capability, which is in turn determined by its maximum thrust and how long this can be maintained - as well as how deep it can go within the gravitational field of the black hole . Gandalf61 (talk) 08:07, 6 July 2011 (UTC)[reply]
With a supermassive black hole, frame dragging may allow for another kind of slingshot effect. The black hole would need to be rotating for that, of course. A smaller rotating black hole would also have a significant frame dragging effect, of course, but the tidal forces would probably grow too high before you got close enough. --Tango (talk) 12:30, 6 July 2011 (UTC)[reply]
Supermassive black holes would likely still have accretion disks, and would not be safe to navigate. 99.24.223.58 (talk) 18:26, 6 July 2011 (UTC)[reply]

detail about a herb

Clinathanus — Preceding unsigned comment added by 60.53.93.193 (talk) 06:32, 6 July 2011 (UTC)[reply]

Clinathanus isn't coming up with anything. Google proposes Clinanthus, Clianthus, or Clinacanthus. Of these, the last sounds most "herbal" in terms of prominent traditional medicinal uses, at first glance, but please confirm. A high-ranking blog hit describes it as "Scientific name: Clinacanthus nutans, English name: Snake plant, Chinese name: 鰐嘴花, E zui hua, 扭序花, Niu xu hua, 竹節黃, Zhu jie huang, 小接骨, Xiao jie gu, 柔刺草, Rou ci cao, 青箭, Qing jian, 鵝嘴花, E zui hua, 憂遁草, You dun cao, 沙巴蛇草, Sha ba she cao and Malay/Local name: Kitajan, Gendis." Our "snake plant" article isn't this. Clearly some research is needed to sort out if all those Chinese terms really refer to the same thing. But first, is this what you want? And what interests you about it? Wnt (talk) 06:41, 6 July 2011 (UTC)[reply]

This page [13] seems to contain reliable information about this herb. Itsmejudith (talk) 06:51, 6 July 2011 (UTC)[reply]

dark matter

Einstein field equation and other scientists cosmology theories resulted some mathematical and or practical results ,some of them are observed and detected in observation and practice .here we list some of theoretical and practical results :


1)dark matter ==>this seems to be any hiden dynamic ;;;;;;;;;;;;; 2)gravity wave ==>this is mathematical result of formula  ;;;;;;;;;;;;;; 3)dark energy ==>equal first ;;;;;;;;; 4)world expanding ==>observed by Hubel;;;;;;;;;;;;; 5)curving light at edge of sun==>observed by Edington ;;;;;;;;;;;;;; 6)quantom gravity ==>unsolved until now  ;;;;;;;;;;;;;;;;;; 7)forcasting black holes ==>foud detection way by Hawking;;;;;;;;;;;;;;;; 8)and some field equation and unite force theorem ;;;;;;;;;;;;;;;;;;


question :what do you do think about

dark matter?--78.38.28.3 (talk) 13:59, 6 July 2011 (UTC)akbarmohammadzade [reply]

copyvio from http://physics.about.com/od/physics101thebasics/a/fiveproblems.htm removed

— Preceding unsigned comment added by 78.38.28.3 (talk) 14:17, 6 July 2011 (UTC)[reply]

Your question contains a verbatim copy of content from About.com. The inclusion of this violates Wikipedia's copyright policy, so I have removed it. -- Finlay McWalterTalk 14:32, 6 July 2011 (UTC)[reply]


thanks Iwas trying to give that link — Preceding unsigned comment added by 78.38.28.3 (talk) 14:45, 6 July 2011 (UTC)[reply]

Dark matter is probably mostly intermediate mass black holes. Even if at some point in the past it wasn't, whatever it used to be would have been accreted by black holes.[14] 99.24.223.58 (talk) 03:05, 7 July 2011 (UTC)[reply]

the brightness of galaxis such as Olbros paradux make us to enter dark matter to dynamic of univers , may it be intermediate mass black holes effective on galaxies brightness , but total mass of any galaxy conatins all object of its body , although it be any black hole . Here I say again that internal properies of any black hole dos not change its interstellar charastricties . suppose any star comes to be black hole and has planets , its planets will coninue to rotate round it after its exchanging . let I want to have new very hot discuss about this subject :

akbar mohammadzade--78.38.28.3 (talk) 05:19, 7 July 2011 (UTC)[reply]

for this discussion I want any one to consider that dark matter comes from solving some equations to show dynamism of univers objects , and total univers density ,and Olbros night brightness paradox

.a. mohammadzade--78.38.28.3 (talk) 07:15, 7 July 2011 (UTC)[reply]

The reference desks are not intended to be a discussion forum. Did you have a specific factual question about dark matter that is not answered by the article to which you linked? {The poster formerly known as 87.81.230.195} 90.201.110.220 (talk) 09:54, 7 July 2011 (UTC)[reply]

Clarification: t-test and degrees of freedom

I'm referring specifically to the statistical t-test and to a worked example in Wikibooks: >http://en.wikibooks.org/wiki/Statistics/Testing_Data/t-tests> .If every parameter you compute from a sample decreases the degrees of freedom you have, why computing the standard deviation does not? In the example there is a sample of 20 measures. The t-test is worked out, using the sample mean and the sample standard deviation. Why the degrees of freedom are considered to be 19 and not 18? --82.50.23.154 (talk) 14:02, 6 July 2011 (UTC)[reply]

I'm not exactly an expert on statistics, but I believe it may be due to the fact that the standard deviation can't be used to uniquely define one of the values. In other words, if you have a sample of 20 measurements, you can calculate the mean, and then if you take any 19 of the measurements and the mean you can calculate the 20th measurement. However, if you calculate the sample standard deviation from the 20 measurements, then you the first 19 values and try to calculate the 20th, you will find that you can't, as the solution will contain a +/- from the square root.
This is only my guess though; it's a very good question.-RunningOnBrains(talk) 18:03, 6 July 2011 (UTC)[reply]

This might be a more appropriate question for the math reference desk. I'm not sure the Wikibooks example gives a very good definition of Degrees of freedom (statistics). You might be better off with that article or Standard deviation#Estimation, which explains why you use N as a denominator for a sample, and N-1 as the same denominator for the population. 99.24.223.58 (talk) 06:34, 7 July 2011 (UTC)[reply]

solar corona

During a total solar eclipse, the solar corona can be seen with the naked eye.solar corona is most hotter than sun surface , what is the reason? has sun corona any special gravity firld , to cause special condition on it?

--78.38.28.3 (talk) 14:41, 6 July 2011 (UTC)A.mohammadzade[reply]

Using the most conventional method of temperature measurement, the corona is hotter than the interior of the sun. For the most part, this is because the plasma is so sparse. Effectively, this means there is a very low thermal conductivity. Equivalently, this means that the mean free path is very large, and the probability of a collision (or any interaction) between particles is lower in the solar corona than it is inside the interior of the sun (say, in the photosphere or in the convective zone). The energy density is lower in the corona, but the corresponding mass density is very much lower, compared to the convective region of the sun. Therefore, the energy per particle is higher in the corona. Most conventional definitions of temperature will therefore resolve a "higher" temperature in the corona. As discussed above, for a sparse plasma, temperature can be defined in more complicated ways, such as defining the energy partitions per specie. Personally, I do not think that temperature is a very helpful parameter when describing plasmas, because it encourages us to imagine the plasma behaving as a hot gas. Nonetheless, most plasma physicists use temperature (with various, specific definitions) as one of the primary defining characteristics of a plasma. I would prefer to see the energy per particle discussed and measured in electron Volts; or simply work with the plasma frequency, which tells us about the specie and energy, and is a more direct measurement anyway. Nimur (talk) 15:32, 6 July 2011 (UTC)[reply]
To sort of "dumb down" what Nimur is explaining a bit (in case it went over your head), the very simplistic way to look at temperature is as a measure of the amount of kinetic energy the particles that make up a substance has. All other things being equal (and you need to assume that all other things are really equal) that means that a higher temperature equates to particles moving at a faster speed. Keep that in mind (its a massive approximation, but it works for this purpose): Higher temperature = faster moving particles. When you get to the corona part of the sun, the particles are so spread out that they can move very far without hitting another particle; since the particles can only slow down by transfering some energy to another particle, that means that particles in the corona don't often get the chance to slow down. Since they can't be slowed down by anything, they just keep moving very fast; that means that the corona has a very high temperature. The same thing happens on Earth, see thermosphere, which is a part of the Earth's outer atmosphere which has a very high nominal temperature, measured in the thousands of degrees celsius. However, if you were floating, unprotected, in the thermosphere, with its several-thousand-degrees temperature, you would freeze to death. Why? Because you are always losing heat via radiation, and you will continue to do so in the thermosphere. The "air" in the thermospehere is so sparse, that even though the particles are moving VERY fast (high temperature) so few of them hit you that they cannot warm you up. --Jayron32 17:27, 6 July 2011 (UTC)[reply]
While the two answers above are essentially correct, they dodge the question which is "Why is the temperature so high?" Dauto (talk) 20:05, 6 July 2011 (UTC)[reply]
That's because no-one is entirely sure. See Corona#Coronal heating problem. --Tango (talk) 22:07, 6 July 2011 (UTC)[reply]
I have asked a few friends who are solar physicists. The most promising lead I have so far is this paper from UCLA: Observations of the Solar Wind... with the WIND 3D Plasma Instrument (1996), from the Berkeley SSL. "...At coronal temperatures electrons are not gravitationally bound while protons are..." - so, as the OP has suggested, there's an electromagnetic effect interacting with a gravitational effect. Electrons are able to behave as a free plasma, while protons can not; doubtless, this creates an non-equilibrium condition, where orbital protons are effectively dragging electrons in directions that they do not want to go. I would not be surprised if the result is an exchange of energy, effectively raising the electron temperature. Solar physics is hard stuff, and the data we have is limited; sorry that there are few simple answers. Nimur (talk) 01:04, 7 July 2011 (UTC)[reply]

Giant jellyfish

This picture turns up on a lot of jellyfish-related searches. Is it real? And if so, what species? I know it is not a lion's mane jellyfish, it looks more like a nomura's jellyfish. But even for a nomura's that is really huge. So what is this thing? --T H F S W (T · C · E) 18:11, 6 July 2011 (UTC)[reply]

It's a photoshopped fake. This question turned up at ref desk in May of 2010.[15]Baseball Bugs What's up, Doc? carrots19:21, 6 July 2011 (UTC)[reply]
Too bad! It would be neat to have a real jellyfish that big. --T H F S W (T · C · E) 01:02, 7 July 2011 (UTC)[reply]
Yeh, keep one like that in your backyard swimming pool, and it would tend to discourage neighbors from dropping by unannounced. ←Baseball Bugs What's up, Doc? carrots11:24, 7 July 2011 (UTC)[reply]

Pyrite problem

I recently purchased pyrite. It has the "fool's gold" look and does not dissolve in HCl like pure FeS does. What is the problem? --Chemicalinterest (talk) 20:20, 6 July 2011 (UTC)[reply]

I thought pyrites, which is FeS2 was insoluble in HCl (and Rutley's Elements of Mineralogy shares my opinion). DuncanHill (talk) 20:26, 6 July 2011 (UTC)[reply]
I guess there's no problem. --Chemicalinterest (talk) 22:21, 6 July 2011 (UTC)[reply]

July 7

Wood catching fire from propane or natural gas

How hard is to catch a wooden structure on fire from a propane or natural gas flash (say, one from a barbecue)? I would imagine pretty hard since it is gone very fast, the heat rises and as a gas it doesn't carry nearly as much heat for the same area as a liquid or solid would, but I don't know. Any insights? --T H F S W (T · C · E) 00:10, 7 July 2011 (UTC)[reply]

It depends on the surface area and moisture of the wood, along with the length of exposure. Most pieces of wood that can fit on barbecues can be ignited by them, but healthy twigs six feet above a barbecue are unlikely to catch fire unless a lot of exploding grease or the like is involved. 99.24.223.58 (talk) 00:19, 7 July 2011 (UTC)[reply]
I don't mean if you've got a piece of wood sitting above a barbecue, I mean from the sudden flash if the gas is left running with the lid closed too long. --T H F S W (T · C · E) 01:01, 7 July 2011 (UTC)[reply]
If you filled a cubic meter of space with propane until something ignited it, whether it could ignite adjacent wood would depend more on what actually ends up igniting it. If there is a source of ignition, that is something to mitigate. If your barbecue is leaking, then you need to mitigate that or you will always be out of gas or pay too much for your gas bill. You need to describe the situational geometry and surroundings of the grill to get a better answer. 99.24.223.58 (talk) 01:53, 7 July 2011 (UTC)[reply]
The structure would be unlikely to ignite directly, but there could be something else that does catch fire, like clothes on an attached clothes line, that then provides the sustained flame needed to light the structure on fire. StuRat (talk) 07:21, 7 July 2011 (UTC)[reply]

Pollination prospects

"One-third of the honeybee population has died off every year since 2006, and evidence points to pesticides used on corn, soy, and wheat crops as the culprits."[16] True? What is the anticipated effect on fruit and vegetable production? 99.24.223.58 (talk) 01:49, 7 July 2011 (UTC)[reply]

You may find the article Colony collapse disorder interesting. --Jayron32 01:54, 7 July 2011 (UTC)[reply]
If bees all went extinct (which is quite unlikely), then we would need to rely on other pollinators, like hummingbirds and humans, or move to crops which don't require pollination. StuRat (talk) 07:17, 7 July 2011 (UTC)[reply]
The pesticides used on corn (i.e. maize), at least, are intended to kill the larvae of corn borers and rootworms; and more and more often those pesticides are being generated from within the plants, as a result of genetic modification. Maize relies on wind pollination, so bees wouldn't be a factor. Wheat and soybeans can self-pollinate. The primary impact from loss of bees would be on fruits and vegetables. ←Baseball Bugs What's up, Doc? carrots11:20, 7 July 2011 (UTC)[reply]
It's rather telling that the original link is by a guy who apparently signed a petition without bothering to see if the claims were true or not. Pesticides have been used for generations. What's special about 2006? ←Baseball Bugs What's up, Doc? carrots11:22, 7 July 2011 (UTC)[reply]
I am ambivalent about this. On one hand I wonder if it is an opportunity to control pollination, which is otherwise a random process, and at the same time reduce the probability of accidental bee stings, which I'd love to see happen. On the other hand, I'm not sure what I would do with control of pollination. Probably more brazil nuts. I'm not sure what most people would do with control of pollination, but I'm not sure it would be better than what bees have been doing. When I was in elementary school the "africanized killer bee sweeping north from Mexico" was the number one impending environmental threat. (Ha! Take that! In your face, africanized killer bees! You all better start making better brazil nuts or I'm cutting my honey budget even further.) 99.24.223.58 (talk) 17:56, 8 July 2011 (UTC)[reply]

Most commons monkeys

What are the most common monkeys known to humans? --111Engo (talk) 02:55, 7 July 2011 (UTC)[reply]

I would guess the Rhesus Macaque but I am not seeing a good reference yet. Rmhermen (talk) 03:22, 7 July 2011 (UTC)[reply]
If you use the term 'monkey' loosely, I'd suggest that you look in a mirror. AndyTheGrump (talk) 03:33, 7 July 2011 (UTC)[reply]
Um, you are not monkey, you are an ape. No joke, please be scientific. :) --111Engo (talk) 03:41, 7 July 2011 (UTC)[reply]
Not necessarily. ←Baseball Bugs What's up, Doc? carrots11:26, 7 July 2011 (UTC)[reply]
Also, humans are not apes, either. We are primates. ←Baseball Bugs What's up, Doc? carrots11:27, 7 July 2011 (UTC)[reply]
Well, it depends upon how you define "ape", but if you take it to be equivalent to a monophyletic taxonomic grouping, then yes, humans are apes (and apes are a subset of primates). The only way humans aren't considered apes is if you define the term "ape" to be a paraphyletic group explicitly excluding humans (which is not common in scientific circles), or completely reject scientific consensus of human evolution. -- 174.31.204.164 (talk) 15:26, 7 July 2011 (UTC)[reply]
Well he did say loosely. More importantly, if you want to be scientific, monkeys are a paraphyletic group. So from a scientific view, it's questionable if it makes sense to ask a question where the answer can be either in parvorder Platyrrhini or superfamily Cercopithecidae or but not superfamily Hominoidea.... Nil Einne (talk) 04:30, 7 July 2011 (UTC)[reply]
Yup. 'monkey' is a vague term, so I gave a vague answer. (And 'ape' isn't exactly clear either, though I'll suggest that whatever they are, we are too...). AndyTheGrump (talk) 14:09, 7 July 2011 (UTC)[reply]

Ok, will anyone go to the original question? --111Engo (talk) 12:19, 7 July 2011 (UTC)[reply]

For the most "commons monkeys", check out wikimedia commons.[17] For the most "common monkeys"... first, define what you mean by "common". Most populous? Most often used in circus acts? ←Baseball Bugs What's up, Doc? carrots12:27, 7 July 2011 (UTC)[reply]
I'd assume 'most populous', i.e. which species of Platyrrhini or Cercopithecidae (and not Hominoidea) has the largest total living population. Even this may be difficult to answer, given that large populations tend to be genetically diverse, and it may not always be clear whether they are all the same species. And then there is observer bias - generally speaking, primatologists tend to be more interested in counting rare species than common ones. AndyTheGrump (talk) 14:09, 7 July 2011 (UTC)[reply]
On that basis the Vervet monkey of East and Southern Africa is possibly also a contender. Roger (talk) 14:14, 7 July 2011 (UTC)[reply]
I should think this question would be a no-brainer.[18] Bus stop (talk) 14:23, 7 July 2011 (UTC)[reply]

Clarification: By "most common", I mean monkeys which are known to most people i.e. monkeys about which most people have heard of and monkeys humans interact with most. --111Engo (talk) 14:27, 7 July 2011 (UTC)[reply]

I think you are asking what is the quintessential monkey—is that correct? As in—what is the iconic monkey, or what monkey comes to most people's minds when they hear the word monkey. If that is what you are asking, I think that different sorts of monkeys would come to mind, depending on which human group were queried. In a society bombarded by media images, I think the most common one is the chimpanzee. Bus stop (talk) 14:44, 7 July 2011 (UTC)[reply]
... which is actually an ape, not a monkey. Gandalf61 (talk) 15:00, 7 July 2011 (UTC)[reply]
In that case the clear winner is the Rhesus Macaque as hundreds of millions of Asian people are familiar with them. Many people interact with them on a practically a daily basis - particularly in India. Roger (talk) 14:53, 7 July 2011 (UTC)[reply]

Year the Duwamish Railroad Bridge in Seattle was built?

I a attempting to finish off a list and create an article on a bridge in Seattle. I have found some in-depth info from engineering journals through Google Books searches, but I am struggling to find a year it was built and think some more info could be useful to create an article. Some keywords and other information that might assist in any searches:

  • One of the earliest "heel trunnion" bridges
  • Operated by Northern Pacific Railway (I think it is now operated by Burlington Northern)
  • Built around 1912
  • Seattle landmark (I cannot find an ordinance #) but not NHRP

Any assistance would be appreciated.Cptnono (talk) 03:55, 7 July 2011 (UTC)[reply]

BirAli Crater

I've visited this area and uploaded the photo here. How can I know if this were an impact crater, (see also the other one beside on Google maps)?--Almuhammedi (talk) 04:45, 7 July 2011 (UTC)[reply]

You can search on the web -- every mention says that it is volcanic. The fact that it is not perfectly round (as you see on the map) also means it is pretty surely not an impact crater. Looking around on the map, there are a number of clear volcanic cones in the area -- you can see one in the background of your photo. The second crater is much more difficult to recognize based on shape, but since there are so many volcanic craters and cones around, it must surely be volcanic as well. Looie496 (talk) 05:20, 7 July 2011 (UTC)[reply]
Then can I consider it as a volcanic crater lake? Can I also add it to the list of volcanic crater lakes? --Almuhammedi (talk) 13:24, 7 July 2011 (UTC)[reply]
Well, I saw at least a dozen sites that identify it as volcanic, but none of them was a reliable source. It would be good to find a book or other published source for the information before adding it to an article. Looie496 (talk) 16:30, 7 July 2011 (UTC)[reply]
The lake is Shouran lake according to Wikimapia here [19]. It looks like a maar, formed by an explosive phreatomagmatic eruption, but that's pure OR. Mikenorton (talk) 22:10, 7 July 2011 (UTC)[reply]
Try one of the following links on Google Scholar: Metasomatism of the shallow mantle beneath Yemen by the Afar plume—Implications for mantle plumes, flood volcanism, and intraplate volcanism (ABSTRACT) or Clinopyroxene-rich lherzolite xenoliths from Bir Ali, Yemen—possible product of peridotite/melt reactions (PDF, 6 pg.). ~AH1 (discuss!) 16:46, 8 July 2011 (UTC)[reply]

Why not adopt measurement scales better suited for very large quantities?

I find that I can imagine the scale of some colossal entities if they are given in large but familiar units, even where those units are Olympic pools of water, or elephants. What is the use of telling me that a large ocean has so many litres in it? They might as well use thimble-fulls. And you get figures like 10 ^ 17, which mean nothing to anyone.

My prime target for amendment would be using miles or kilometres for astronomical distances. I remember a mile as being from my back fence to Mrs Carruther's chook pen. It's a short walk. We use light years for very large distances - why not use them for distances in our local solar system. For example, it is about 8 light minutes to the sun, and about 1.5 light seconds to the moon. This gives one a pretty good idea of the relative distance of the sun and moon from Earth. The distances to Jupiter and the other gas giants are about 4 to 8 hours.

Voyager 1, launched over 30 years ago is the furthest man-made object from Earth. The NASA site http://voyager.jpl.nasa.gov/ gives it as being 17,481,723,800 kilometers away, a figure which is billons of times further than Mrs Carruthers chook pen - and means nothing to most people. It also gives the distance as 117 AU (Astronomical Units - a single one being the distance between the Earth and the Sun.) Now, if we accept that 1 AU is about 8 light minutes, then Voyager 1 is about 15.5 light hours away, or nearly double the distance to Jupiter. The Sun is 8 minutes away, and Voyager is 15.6 hours away. Hey, suddenly everyone can see these distances in perspective! This is just common sense. Why doesn't NASA use this instead of AU's and kilometers? And the speed of light is totally constant too, an additional plus.

Same goes for Olympic swimming pools. Everyone has been in one and has a good idea of how much water they hold. If you had a unit "Kilopool", i.e. a thousand swimming pools, you could express the scale of very large quantities of liquid in a way that a person could easily visualise. For example, a Kilopool would be a cube, with each side having 10 Olympic pools. In my view, the whole system of measuring large quantities in thimblefuls and feathers and then announcing that something is 10^17 is absurd, and we should adopt scales like the ones I've suggeted here. It's a no-brainer really, or I should say, a one-pea brainer.

I blame the frogs (the French) for starting this. The centimetre was apparenty picked as a basic lengh unit because it did not naturally measure anything, unlike the inch, which is about the length of man's thumb. The metre is too long to be a step, too short to be a room length, and so on.

Now, watch, the Kilopool idea will suddenly and quite by "coincidence" be put forward as an official unit. And will I get a jot of recognition? No sir! I'm going to put on my alfoil thought blocking cap right now. Myles325a (talk) 04:49, 7 July 2011 (UTC)[reply]

If you don't mind removing your tin foil hat for a minute...
You seem to be ranting about the lack of using measurements which result in small numbers. I am with you on this matter; for some godforsaken reason the astronomical community continues to use CGS units. I always found it ironic that the people who study the largest and heaviest objects in the universe use smaller units than the kilograms and meters that most scientists use. I believe in using units that result in small numbers for the scales you are using, although not to the point of making unit conversions awkward.
However, you have to see the conundrum that we scientists have when communicating to the general public: if we use crazy units like megaparsecs, light minutes, and AUs, Joe Shmoe has no idea what those are. Kilometers, as you say, are about 60% of the walk from your back fence to Mrs Carruther's chook pen, which was a short, easy walk, and an easy distance for you to comprehend. Could you tell me how long it would take to walk a parsec? To me, the best solution is to give both a normal unit (miles) and a small-number unit (parsecs or what have you), which would satisfy both crowds. And contrary to your assertion that NASA does not do this, on this page they list Voyager 1 as being "16.9 billion kilometers (~ 113 AU)" from Earth.
Additionally, if you look at meter, you can see that it was originally defined as one ten-millionth of the distance from pole-to-pole, to avoid using arbitrary objects to define the measurement like "foot". Having a standard unit of measurement was useful for science. Now if only we could convince those damn Americans to use it. You can put your foil hat back on now :) -RunningOnBrains(talk) 05:08, 7 July 2011 (UTC)[reply]
The metre is actually one ten-millionth of (North) pole to equator. AndrewWTaylor (talk) 08:03, 7 July 2011 (UTC) [reply]
I have struck one of my previous comments, since apparently I need to learn how to read, but this leads me to a further point: why do you think light years or light minutes are easier for the public to understand than an Astronomical Unit (AU)? To me, AUs make more sense in the solar system, especially because we're often talking about distance from the sun, so when I see that Voyager is 117 AUs from the sun, I think "hm, so it's more than 100 times further away from the sun than we are!". To me that's much more understandable than "light minutes", but it's all personal preference.
I don't know about you, but I'm sure that not every one has been in (or even seen) an Olympic size swimming pool. I certainly haven't, at least not recently. And no scientist that I know of actually uses this measure. Your idea for using pool-length for volume makes no sense, since these pools have different length, width, and depth, so it wouldn't really be even on each side (they aren't a standardized depth anyway: if you read the link, they only have to be a minimum depth). Vulcanologists and oceanographers deal with extremely large volumes all the time, and so they use units like cubic miles and cubic kilometers (and my favorite-sounding one: Sverdrups). Why aren't these acceptable? As I said above, you can't really satisfy everybody at once.
I'm personally a fan of Rhode Islands as a unit of measure for area. Which do you prefer?-RunningOnBrains(talk) 06:05, 7 July 2011 (UTC)[reply]

OPmyles325a back live. Hi RunningOnBrains, I was writing a hefty reply to your note when a glitch occurred and I lost the lot. When I got back, you had made another entry. Thanks for the effort. As I wrote initially, I can understand kilometers and miles very well, because I experience such directly every day from childhood, as I had earlier supposed everyone else did. My objection to miles / kays being used for astronomical distances is that the familiarity we have with these distances is drowned out by the sheer number of the miles/kays involved. I don’t know where the cutoff point is, but I strongly suspect that billion is about the farthest an ordinary human mind can take in. The problems with AU’s are twofold. First, most laypeople have little notion of how far the Earth is from the Sun (or the Moon for that matter). The value of AU is depleted if many think Earth /Sun distance is a matter of thousands or trillions of miles / kays. The second problem is that AU is only good for the Solar System and really only the inner Solar System at that. Once you get into interstellar space and then intergalactic space, AU’s become very much like miles / kays – there are just too many of them to get a grip.

The great thing about light speed is that it is based on time, which humans experience directly and fundamentally. EVERYONE knows what 10 minutes is, and also 10 years. If a driver asks for directions and you tell him that a warehouse is “10 minutes yonder”, then you will never get a request for clarification. It’s as clear as can be. That’s because he knows the normal speed of a car, and he knows what 10 minutes is. You can tell someone that light could travel around the equator 7 times in a second. Anyone who has an atlas will take that on board. Now if you tell them that the Moon is about 1.5 seconds away, he can see that means that it is about 11 Earth circumferences away. And if you tell him that the Sun is 8 minutes away, he can compare that with the distance from the Moon, and start to get a very good idea of the general nature of where we are. This is not the case when you start rabbiting on about “billions of miles” and so on. And as I said initially, you can then proceed to tell him that the gas giant planets are about 4 to 8 hours away, and Voyager is about 15 hours away. What could be simpler? It is a matter of taking what the reader is well familiar with, and then building on that, but not in a way where suddenly you expect him to visualize figures with 15 zeros in them.

Once we get beyond the planets, the system still works well. Anyone who has done some history will have a good idea of what “a thousand years” means. After all, people can live to be a hundred, and a thousand is no more than 10 of them, with one born as the other reaches 100. Galactic distances are amenable to figures no larger than thousands of light years, so we are in a position to comprehend them with relative ease. Try telling someone how far we are from the galactic centre in AU’s - it might as well be centimeters. So I cannot fathom why the NASA site - and thousands like it - don’t permanently ditch this ridiculous, and artificial, feathers and thimbles approach, and use the superb scale that has been provided for us in the constant ruler of light. I was surprised to hear you say that you had never been in, peed in, or been seen in, an Olympic swimming pool. What, have you never even watched a swimming event at the Olympics? Not to worry, we could have a base unit of a Wading Pool. I guess being an Aussie, I just assumed that everyone spent summer diving and cavorting in large pools. Yes, a flaw with the Kilopool idea is that an Olympic Pool is rectangular, not square, and it is deep at one end and shallow at the other. But these flaws are not serious. The important thing is that from early childhood, an Aussie kid is up and down the lanes in the pool, dives down to the bottom, and thus gets a very good idea of how much water there is, not only with his or her mind, but with every inch of their bodies.

In the case of employing units which, as you put well, give small and comprehensible numbers to describe large quantities, it is necessary to use entities which are well-known to those likely to come across them and be interested in them. So it appears that I have been guilty of cultural imperialism when I suggested the Kilopool. In much the same way as using "Rhode islands" as a base is culturally insensitive, because outside of the U.S. few people would know how big that means. (On an aside, it is interesting to see how often Belgium is used to denote large areas, because Belgium is quite small, probably smaller than most people realize.)

The reason that elephants are often used for large weight concerns is that writers are assuming that people have seen them in zoos, and have a rough idea of how heavy they would be. Blue whales are much heavier, but then they float and that massive weight is not so evident. Myles325a (talk) 07:00, 7 July 2011 (UTC)[reply]

Ask yourself how accurately in percent you could establish, maintain and transfer to others your proposed pool-length unit. Then see the article Metre and compare. Then please look into the usefulness of the Logarithmic scale which is a good way of managing very small and very large quantities that are inevitably beyond tangible comprehension. In some cases such as sound amplitude and sound frequencies in music the log scale actually agrees better with human experience than a simple linear scale. Please don't call the citizens of France "frogs". They are a proud nation with nuclear capability and veto right at the UN so it is unwise to get them hopping mad. See Australia–France relations. Cuddlyable3 (talk) 08:23, 7 July 2011 (UTC)[reply]
I have a better idea. Why don't educate the public better so they can understand what a power like 10 ^ 17 mean? Dauto (talk) 17:49, 7 July 2011 (UTC)[reply]
What does 10^17 mean to you Dauto? It is a difficult number to conceptualize. Think of it this way, if 10^17 grains of rice were given to each person on earth, that would be around 1000 lbs per person. That much rice would fill enough grain cars to make a train long enough to circle the earth 13 times. Education can only go so far when it comes to conceptualizing incredibly large numbers. Googlemeister (talk) 18:50, 7 July 2011 (UTC)[reply]
I think you did a pretty good job at conceptualizing it proving my point, thank you. Dauto (talk) 19:21, 7 July 2011 (UTC)[reply]
Granted, but it took me 8 minutes, a graphing calculator and looking up the size of a railroad hopper car to do it. Googlemeister (talk) 14:09, 8 July 2011 (UTC)[reply]
A 60-bit binary computer can keep track of every grain of rice through a calculation without losing or gaining a single grain. Conceptualise that. Cuddlyable3 (talk) 20:03, 7 July 2011 (UTC)[reply]
The ocean volume is often measured in cubic kilometres, rather than in mass units of "1.024 ± ~0.005 x 109 metric tonnes s.v.". ~AH1 (discuss!) 16:37, 8 July 2011 (UTC)[reply]

Species identification request

Hello, I was wondering if anybody could help me identify a few species of insects and plants before I upload the pictures to Commons. There are three different species of plants, one with pink flowers, one with white and pink flowers, and one with white flowers. There are also two species of dragonfly, one which is bright green and black (see pictures here, here, and here) and a yellow and black one. All pictures except the first were taken at Tirta Gangga lake in East Lampung Regency, Lampung, Sumatra while the first picture was taken near the regent's office in Sukadana, Lampung. Any help would be greatly appreciated. Crisco 1492 (talk) 05:03, 7 July 2011 (UTC)[reply]

Sean.hoyland - talk 11:31, 7 July 2011 (UTC)[reply]
Would Plumeria rubra be the best guess for the one with white and pink flowers? Crisco 1492 (talk) 15:20, 7 July 2011 (UTC)[reply]
Yes, I guess from the leaf shape but there are many named cultivars. Sean.hoyland - talk 15:45, 7 July 2011 (UTC)[reply]
The joys of being a botanist. I have uploaded the Plumeria at File:Plumeria at Tirta Gangga, Sumatra.jpg, but we have many, many, many ad infinitum better pictures of Bougainvillea. So one plant and two dragonflies to go. Crisco 1492 (talk) 16:01, 7 July 2011 (UTC)[reply]

what will happen If any massive star comes to be black hole and it has planets?

One example of the complex planetary interactions in post-supernova star systems. ~AH1 (discuss!) 16:17, 8 July 2011 (UTC)[reply]

A.mohammadzade--78.38.28.3 (talk) 06:22, 7 July 2011 (UTC)[reply]

While I don't see any reason planets couldn't have stable orbits around a black hole, they couldn't remain after a star has a supernova explosion leaving a black hole, because the portion of the supernova which blasts outward would destroy any planets there. However, there might be some complex multiple-body gravitational interaction which brings new planets into stable orbits about the black hole, later on. StuRat (talk) 06:34, 7 July 2011 (UTC)[reply]
A supernova explosion would not necessarily disperse a planetary system completely; see PSR B1257+12 for an example of a planetary system around a neutron star, which is the end result of some supernovae.-RunningOnBrains(talk) 07:05, 7 July 2011 (UTC)[reply]
Yes, but the concern here is the larger supernovae which produce black holes. Would any planets survive those ? StuRat (talk) 08:16, 7 July 2011 (UTC)[reply]
The back of my envelope suggests that a supernova is probably survivable for large and distant planets, provided one is only asking whether the planet receives less energy than its gravitational binding energy when exposed to a core collapse supernova. A large fraction of the planet would still be completely ablated, but the event might be survivable (provided survivable merely asks whether any gravitationally bound nugget still exists). Dragons flight (talk) 09:11, 7 July 2011 (UTC)[reply]
Hmmm, I though that the planets of PSR B1257+12 were formed after the neutron star formed... Count Iblis (talk) 14:32, 7 July 2011 (UTC)[reply]

I abslotly think that the black hole (not super nova) cannot change condition and rotational orbit of planets round early star except one of them come to roche limitMohammadzade--it means that if any star had planets and end its energy to be black hole those planets still rotate round main star 78.38.28.3 (talk) 06:41, 7 July 2011 (UTC)[reply]

Consider that all exchangings are happening inside the event horison.and consider the binary star systems which first detected black holes --

78.38.28.3 (talk) 06:58, 7 July 2011 (UTC)A.mohammadzade[reply]

Let's imagine that somehow a planet is shielded from the supernova of its host star and survives that star becoming a black hole. As the star has lost mass in the explosion, its overall gravitational field is diluted, and the planet would move away from the Black Hole, and its orbit would be longer. I would imagine, the orbit would be further altered as the Hole's gravitational attraction comes from a point, not a finite area. The forces involved in the preservation of angular momentum would thus become intensified, and if the planet did not face the Sun all the time before, it might do so now. Also, it would now be very dark on the planet, and life would be much harder. If there was an intelligent and technological species there, they might be able to survive, using the energy released by the tidal forces of the Black Hole on their home planet, and living near hot vents and volcanos and the like. Or alternatively, they might exploit the Black Hole by sending material streaming into it, and collecting the energy dissipated when the Hole rips it apart and spaghettifies it. If they have sophisticated control over genetic processes, they might biologically engineer their species to be only an inch or so high, with work to be done by robots. That way, they would need very little food or fuel, and could live happily on the smell of an oily rag. Myles325a (talk) 07:16, 7 July 2011 (UTC)[reply]

please dont mix supernova with black hole. --78.38.28.3 (talk) 09:30, 7 July 2011 (UTC)[reply]
Do you have another mechanism for forming a black hole that does not involve a supernova? --Jayron32 19:52, 7 July 2011 (UTC)[reply]
The big bang likely made primordial black holes, such as the supermassive one at the center of our galaxy. That could explain dark matter with intermediate mass black holes and according to Lacki and Beacom (2010) that would explain why people haven't found any weakly interacting massive particles. 99.24.223.58 (talk) 20:39, 8 July 2011 (UTC)[reply]
There may be some minor relativistic effects, but the shell theorem tells us that (with Newtonian gravity, which is a good approximation at the distances we're talking about) there is no difference between the gravitational field of a point-mass and the gravitational field of a spherically symmetric object (which a star is, give or take an equatorial bulge). I don't understand your comment about tidal locking. If gravity is reduced, then so are tidal forces. They are also reduced by the increased distance you mention. The chance of becoming tidally locked would reduce (or, perhaps more accurately, the time it would take to become tidally locked would increase). In order for the civilization you describe to exploit the block hole, they would need to get closer to it than the original radius of the star, since outside that radius nothing has really changed. That won't be easy. Geothermal energy would work for a time, although the planet will cool down quicker without a star keeping it warm (although that could still be millions of years, I'd need to look up some numbers). --Tango (talk) 12:03, 8 July 2011 (UTC)[reply]

The supernove makes neutron star and remnant nebula ,any star which changes to black hole , condences to Schwartz shieldradius and matter disapiers in any point , that dosenot through the matter to interstellar space , this is diffrence between neutrone star and black hole , neutron star or pulsar is not same to black hole and it send pulses .Ilast said this sentences about them:

I love supernovae for their first brightness as some people suppose them new star , their palpitation such as our heart , and their expanding such as universe , and their rule in our life with sending material of our body" A. mohammadzade somebody used to say something about black holes , I prefer to say about supernovae , the first one is death and being secretary , the second one shows celebrating and brightness , and life .A. mohammadzade — Preceding unsigned comment added by 81.12.40.120 (talk) 20:29, 8 July 2011 (UTC)[reply]

Estrous vs Menstruation

I've read the Estrous cycle article, and the Menstruation article. The latter is primarily focused on humans, to my disappointment. I have a few questions:

Which appeared first, estrous or menstruation? Do more mammals go through estrous or go through menstruation? Is estrous equally powerful across all species that have it? (example: my female cats became mindless horny robots for ~10 day periods every month or so before they were spayed) Apart from humans, does menstruation not have as strong a mental effect? Why?

Thanks for your help! The Masked Booby (talk) 06:10, 7 July 2011 (UTC)[reply]

I fixed your Estrous cycle link which was not working. Cuddlyable3 (talk) 07:42, 7 July 2011 (UTC)[reply]

After spending the better part of a half hour chasing this, I came up with the following which I added to menstruation: "Though there is some disagreement in definitions between sources, menstruation is generally limited to primates. It is common in simians including Old World monkeys and apes and New World monkey, but variably expressed in prosimians, being completely lacking in strepsirrhine primates and possibly weakly present in tarsiers. Outside the primates it is known only in bats and the elephant shrew, an insectivore.[1][2][3][4]" I didn't comment on a controversy, running back to 1898, about whether the elephant shrew isn't really menstruating because ovulation occurs at that time (or in a low-grade online source I found, shortly afterward)[20] Note that aside from the elephant shrew - doubly ambiguous because its taxonomy has also been disputed - all of the menstruators appear to be in Archonta. Wnt (talk) 18:56, 7 July 2011 (UTC)[reply]

checking xray escape from baggage machine

A friend works at an airport, and the other day a passenger had a geiger counter with them them to observe the increased radiation at altitude. He let the staff put it through the xray security machine and it showed a higher reading, no surprise. but then when it was held at the conveyer belt it showed a fluctuating reading when the curtains were moved by baggage coming through.

What device can he use to detect if xrays are getting out? would a digital camera (which detects infrared) also be useful? It would seem that any xrays getting at people would be bad. Polypipe Wrangler (talk) 06:11, 7 July 2011 (UTC)[reply]

He seems to have the right device. If you wanted a movie showing it, you would need a fluoroscope screen. Note that the main danger is to the security guards who stand there all day long, and might want to wear protective clothing. You didn't ask, but the two cures I can think of are either to stop the belt, close the curtains, use the X-ray, then turn the X-rays off and restart the belt, or to make the belt longer with multiple sets of lead curtains. StuRat (talk) 06:37, 7 July 2011 (UTC)[reply]
A Geiger counter responds to any kind of ionising radiation so what came through the curtains may not have been xrays. One can speculate what kind of radioactive trace someone's baggage may have left on the conveyer. I would expect a rubber curtain to be transparent to xrays but it might also act as a diffraction grid. Cuddlyable3 (talk) 07:39, 7 July 2011 (UTC)[reply]
Are they rubber curtains ? What would be the point in those ? I assumed they contained lead or some other heavy element, to act as an X-ray shield. StuRat (talk) 08:12, 7 July 2011 (UTC)[reply]
Yes I was going to say a similar thing, [21] suggests they contain lead. Nil Einne (talk) 08:15, 7 July 2011 (UTC)[reply]
Curtains like these? Cuddlyable3 (talk) 08:35, 7 July 2011 (UTC)[reply]
That photo is of a baggage claim conveyor, not the X-ray machine. The baggage claim conveyor typically connects to some outside area. The rubber curtains are just to keep the cold/hot air out of the building. Rckrone (talk) 15:13, 7 July 2011 (UTC)[reply]
The proper test for this would be to require TSA employees to wear dosimeters. 99.24.223.58 (talk) 18:42, 8 July 2011 (UTC)[reply]

Electrical Stunning vs Tasers

Can I ask: Is there any fundamental difference in operation and effect between the electrical stunning techniques used on animals pre-slaughter, and the Taser stun-guns used by various police forces?

If the taser is apparently very painful, is the pre-slaughter technique any less painful for the animals? (I've never been tasered, thankfully). Eliyohub (talk) 12:18, 7 July 2011 (UTC)[reply]

The taser is painful because it doesn't kill. However, it is not nearly as painful as it appears to be. We associate tensed muscles with pain. The electrical current causes the muscles to tense up. So, bystanders assume that there is extreme pain. It is actually far less painful than accidentally touching and discharging a CRT (from experience - the CRT knocked me out cold, but the taser was just a sting). In a slaughterhouse that I visited, cow prods give the cows enough of a sting to make them move. It isn't enough to cause them to completely tense up and hit the floor. Before slaughter, the cows had one metal clip attached to an ear and another to a nostril. Then, an extreme amount of current was sent through the cow's brain. The cows didn't tense up or cry out or anything. They went from looking around to being dead on the floor in an instant. So, it is obvious that both the operation and effect are very different. -- kainaw 12:41, 7 July 2011 (UTC)[reply]

Hardest Known Material

So, lonsdaleite is harder than diamond. But... what is the hardest material known to man? 68.48.123.29 (talk) 13:42, 7 July 2011 (UTC)luos[reply]

Diamond is the hardest confirmed naturally occurring substance, but several attempts (i.e. Aggregated diamond nanorods, Rhenium diboride, etc.) have been made to synthetically form a material that is harder. It is hard to say, however, as many of these reports remain unconfirmed. This article may interest you, as it pertains to some very rare substances (including lonsdaleite) w/ similar structures as diamond, but may be much harder due to various reasons. Tyrol5 [Talk] 14:53, 7 July 2011 (UTC)[reply]
Quarkium. Degenerate matter is the densest material in the Universe, so quarkium (which would be to quark stars what neutronium is to neutron stars) would be the most dense. YMMV as to whether this qualifies as "known to man"? -- SmashTheState (talk) 20:09, 7 July 2011 (UTC)[reply]
Dense yes, but hard? You're sure it's not a superfluid? Wnt (talk) 20:20, 7 July 2011 (UTC)[reply]
If we were to use the Mohs hardness scale, then quarkium would easily outmatch any other substance, since attempting to scratch something that dense would be utterly futile. I don't think its state really matters at that density. Although you may be right, the physical properties of something like quarkium are probably wildly bizarre, and I don't have the scientific knowledge to even guess at it. -- SmashTheState (talk) 21:46, 7 July 2011 (UTC)[reply]
The problem is, the conditions under which quarkium exists are conditions under which any matter will be converted to quarkium. (At least, that's my understanding.) If you got a diamond in contact with the quarkium in order to try and scratch it, you wouldn't have a diamond any more. --Tango (talk) 12:06, 8 July 2011 (UTC)[reply]
Depending on how much quarkium you have on Earth, the energies involved in any material attempting to contact it (or even attempting to hold it?) would likely annihalate the entire planet. ~AH1 (discuss!) 16:06, 8 July 2011 (UTC)[reply]
At low temperatures diamond is harder than boron nitride, assuming an oxygen atmosphere. 99.24.223.58 (talk) 20:52, 8 July 2011 (UTC)[reply]

Third most important blood property

What is the 3rd most important blood property after blood group and Rh?--188.146.90.106 (talk) 16:21, 7 July 2011 (UTC)[reply]

WHAG here but I'd say it was clotting factor. --TammyMoet (talk) 17:16, 7 July 2011 (UTC)[reply]
I don't think the question is valid. Blood group is practically irrelevant except for the practice of blood transfusion from human donors, which was invented only recently and will probably become obsolete just as quickly. Rh creates a notable fertility problem, yes, but it's only our current level of understanding which makes it important to test in advance. Presently perhaps CCR5-Δ32 is the most important trait, at least in Africa. It shifts over time, and in the long run, perhaps everything is of equal importance in evolution, as otherwise the more important part would receive special attention. Wnt (talk) 19:01, 7 July 2011 (UTC)[reply]
THE most important blood property is its role in transporting nutrients and wastes, right? ←Baseball Bugs What's up, Doc? carrots20:18, 7 July 2011 (UTC)[reply]
Oxygen/CO2 transportation is much more important.
Hmmm, are we talking simply about characteristics, or about genetically variable traits? My answer concerned the latter. I suppose blood does have specific "purposes" which are indeed more important than others, but these are things which are genetically non-negotiable. Wnt (talk) 22:38, 7 July 2011 (UTC)[reply]
Since blood group and Rh were mentioned, I presume the question refers to properties that determine compatibility. This may be a homework question, but what the hell, I'll answer it anyway: as our Blood transfusion#Compatibility testing article says, once blood type and Rh have been verified, it is important to screen for antibodies that may react with the donor blood. Looie496 (talk) 22:50, 7 July 2011 (UTC)[reply]
"Screening for antibodies" describes all the tests. ABO blood group and Rh blood group are all about antibodies. A, B and Rh are antigens that the person receiving the transfusion may have antibodies to. If they have those antibodies, then the antibodies will attack the blood. The question, therefore, is which antibodies is it the next most important to screen for. --Tango (talk) 12:09, 8 July 2011 (UTC)[reply]
Neither Cross-matching or Blood transfusion seems to mention which antibodies they are testing for. Rmhermen (talk) 13:36, 8 July 2011 (UTC)[reply]
You don't need to test for specific antibodies. You just mix the donor blood with the receiver's blood and watch for a reaction. It doesn't really make any different which antigen in the donor blood is being reacted to. If there is a reaction, then the blood isn't compatible. --Tango (talk) 22:33, 8 July 2011 (UTC)[reply]
Physiology is an interconnected system. This question is like asking which mole in a Whac-A-Mole game is most likely to pop up next. 99.24.223.58 (talk) 20:53, 8 July 2011 (UTC)[reply]

Astronomical observation time

Atmospheric refraction

Are there any time corrections being made during Moon/Sun and star observations because of incoming light delay, such as that of 8 minutes in case of the Sun?--188.146.90.106 (talk) 16:26, 7 July 2011 (UTC)[reply]

The timings for regular events (i.e. sunset) are based on actual physical sunset, while sunlight (perceived by humans as the sun itself) is still visible for several minutes due to atmospheric refraction (see image to right). For star observations, however, the timings are determined based on predicted position in the sky (sometimes it can take several millions of years for light to reach Earth, enabling us to "see back in time"). For nearby astronomical objects (i.e. the Moon), the time delay (something like two seconds) is negligible and is not taken into account specifically in determining observational timings. Tyrol5 [Talk] 16:44, 7 July 2011 (UTC)[reply]
I was just reading something neat about that topic - the speed of light was first correctly measured by observations of the moons of Jupiter! It never ceases to amaze me how the invention of lenses opened up so many fields of biology and physics to explosive growth all at once. Wnt (talk) 19:12, 7 July 2011 (UTC)[reply]
The thing is, it doesn't take 8 minutes for "just" the light from the sun to reach us, it takes 8 minutes for ANY information from the sun to reach us. So there's nothing to correct for. From earth, the sun IS on the horizon when we "see" it there, not 8 minutes previous, absolutely no part of the sun is there 8 minutes prior, not the gravity, not the light, not the information, not even any mystical magical "sun" force we haven't discovered yet (well i'm not 100% certain about that one, but call it 99.99% certain) For all we know, the sun might have completely vanished a few minutes ago and it would be completely impossible for us to know until that information reached us, at the speed of light. Vespine (talk) 05:00, 8 July 2011 (UTC)[reply]
Let's say the Sun vanishes exactly at sunset from a given position on Earth with an unobscured western horizon and no refraction. Would we fail to notice the Sun disappearing until it is already no longer visible on the horizon? ~AH1 (discuss!) 16:03, 8 July 2011 (UTC)[reply]

Regarding special relativity and time dilation

Let's say Planet A and Planet B are 100 lightdays apart. A spaceship leaves Planet A and heads to Planet B, travelling at about .99 c (from the relative position of Planet A). While travelling, the spaceship broadcasts a song by radio towards each of the planets.
Now, as I understand it, the following will occur:

  • The ship will reach Planet B in about 102 days from the viewpoint of Planet A.
  • The song will be redshifted on Planet A (and thus slowed down), and blueshifted on Planet B (and thus sped up)
  • Time will pass more slowly on the spaceship, relative to the two planets (although, to the spaceship, time seems to pass normally, and everything else is sped up).

So, the questions I have are:

  • How long will the journey take relative to the spaceship? (exact figures aren't necessary, just ballpark)
  • How long will the journey take relative to Planet B? This is what's confusing me the most. So the ship would leave Planet A, and that light would reach Planet B 100 days later, but the ship obviously couldn't then arrive one day later. So time dilation would have the ship move more slowly through time to "correct" this. Err... wait a second, am I answering my own question?

Heh, typing it all out actually made it make more sense. So would it then appear to take about 102 days for the journey from each vantage point (with time dilation affecting the ship)? Also, would the song sent from the ship appear to be the exact same speed to each planet, despite being redshifted for A, and blueshifted for B? I guess my main question is: am I getting this?!
Cheers. --Goodbye Galaxy (talk) 16:30, 7 July 2011 (UTC)[reply]

Not sure about the spaceship's apparent trip duration offhand, but I can answer the others.
  • If Planets A and B are at rest with respect to each other (you don't specifically say this, but I think it's your intent), then each observes the trip to take the same amount of time. 102 days sounds about right, but I haven't done the math.
  • Planet B will in fact only see light from the ship's departure a couple days before the ship actually arrives. This is fairly straightforward, since B observes the ship approaching at .99c, moving just behind the light from its departure.
  • The song will be redshifted towards A and blueshifted towards B. The shifts will be of similar magnitude but opposing direction. Assuming the song is played for the length of the ship's voyage, each planet receives the song over a period of (departure day + transmission lag) to (arrival day + transmission lag). For Planet A, that's Day 0 to Day 202; for Planet B, that's Day 100 to Day 102.
Does that help? — Lomn 18:35, 7 July 2011 (UTC)[reply]
  • The ship takes 101 days (not 102) to reach planet B from the point of view of either planet.
  • Calculating the -factor:
  • The trip takes from the point of view of the ship. Dauto (talk) 18:39, 7 July 2011 (UTC)[reply]
Hmm... both these response have just made me more confused. Lomn's response doesn't seem right, because if B tracked the ship from its departure, it would see the ship travelling 100 lightdays' distance in two days, which is faster than c, and thus impossible.
Dauto's response confuses me, because it seems the ship would observe itself as approaching B faster than c as well. --Goodbye Galaxy (talk) 19:01, 7 July 2011 (UTC)[reply]
The ship sees itself approaching B at 0.99c. That only takes about 14 days because from the point of view of the ship the distance between the planets is Lorentz contracted to about 14 light-days. Lomn is also right because even though the trip takes 100 days from B's point of view, he sees it happening over only 1 day because of Doppler effect. Dauto (talk) 19:06, 7 July 2011 (UTC)[reply]
To be clear, how long the trip takes from B's point of view, and how long it takes him to observe the trip are different because of time delays due to the finite speed of light (B uses light to observe the trip). Dauto (talk) 19:10, 7 July 2011 (UTC)[reply]
Ah, of course. I forgot that distances are relative as well. I'm still slightly confused about Planet B's point of view. I guess I've been operating under the assumption that no object can even appear to move faster than c, but... I was wrong? (p.s. science is awesome, and thanks for your answers, guys) --Goodbye Galaxy (talk) 19:20, 7 July 2011 (UTC)[reply]
See also superluminal jet, a real-world example. Wnt (talk) 19:31, 7 July 2011 (UTC)[reply]
From the perspective of the "moving" observer, there's a cute unit, the "roddenberry", describing how fast it looks like you're going - alas, we don't have an article on it! Wnt (talk) 19:34, 7 July 2011 (UTC)[reply]
The observers on Planet B know the speed of light and will allow for it. Therefore, then they see the ship leave Planet A, they'll know that it actually left 100 days ago. Therefore, when the ship arrives a day later, they know the trip took 101 days (the people on the ship will disagree, of course). --Tango (talk) 12:13, 8 July 2011 (UTC)[reply]

Weak Interaction - article

Hi, reading the weak interaction article, I have remained with some question ? In quantum physics we have learned that always we have two or more particles with electric charge, they interact exchanging photons and so we have electromagnetic force among them. Also, always we have particles very close, in distances lower to proton diameter, we have strong force interacting based in echanging of gluons among quarks. But I have learned in the article that all fundamental particles have weak isospin that acts as electric charge. So I have two questions: 1. - Do quarks exchange bosons (W+, W-, Z) all the time ? and , or 2. - What conditions cause a down quark decay to up quark since we can´t predict this phenomenon ?

Thanks for all, and my comment is based on fact that it is very difficult to get good explanation about weak interaction for begginers. This article can helps a lot. — Preceding unsigned comment added by Futurengineer (talkcontribs) 17:19, 7 July 2011 (UTC)[reply]

1. Yes, quarks do exchange virtual (W+, W-, Z) all the time.
2. The decay is completely random. no special condition is required.
Dauto (talk) 18:29, 7 July 2011 (UTC)[reply]
You may also be interested in strangeness and charm. ~AH1 (discuss!) 15:54, 8 July 2011 (UTC)[reply]

Orbits after central mass changes

Can't quite figure out how to approach this. Some satellites are in circular orbits around the Earth. Suddenly the mass of the Earth doubles. What happens to the orbits of the satellites? Twang (talk) 19:07, 7 July 2011 (UTC)[reply]

It becomes elliptic with the satellite at the apogee. Dauto (talk) 19:15, 7 July 2011 (UTC)[reply]
We could also be more rigorous in defining the behavior if you can elaborate on the meaning of "suddenly." Even if the change is (theoretically) instantaneous, the branch of classical mechanics called continuum mechanics elucidates the mathematical procedures you would need to use to model an instantaneous change of mass, based on where the mass came from and how it distributed itself. I'm thinking about an "accretion process" where the mass accretes on a time-scale much faster than the orbital time-scale. Gauss's law for gravity may also help you identify the conceptual behavior for this thought experiment. Nimur (talk) 20:09, 7 July 2011 (UTC)[reply]

Let's try simpler: Given a planet with mass M, and a satellite in circular orbit with radius r. How to find r for a satellite with the same kinetic energy orbiting a planet with mass 2M. (Actually matter accretion is what got me thinking about it, but not trying to build a computer model, just trying to get a handle on what's involved ... I don't really grok orbital mechanics.)Twang (talk) 22:14, 7 July 2011 (UTC)[reply]

If I recall correctly, a satellite in circular orbit has one-half the kinetic energy it needs to achieve escape velocity, i.e. to reach zero gravitational potential energy. Gravitational potential energy is negative and proportional to m1m2/r. Since the kinetic energy remains the same, the gravitational potential energy needs to be the same, which means if you double m2 you need to double r. So if you double the planet's mass, the satellite needs to be twice as far out to orbit at the same leisurely speed as before. To put this another way, the orbital period of a satellite with a given semi-major axis will decrease according to the square root of the central mass. Wnt (talk) 22:49, 7 July 2011 (UTC)[reply]

Why do so many adult males like my little pony?

Question moved to Wikipedia:Reference desk/Entertainment

Genetics of garden flower colours

Some garden flower species can have a variety of different coloured flowers. I have some Hollyhocks, Alcea rosea, in my garden with each plant having flowers of a particular colour. What are the chances of their seeds producing plants with flowers the same colour? Is anything known about the dominance or recessiveness of the genes for flower colour? 92.28.254.38 (talk) 21:00, 7 July 2011 (UTC)[reply]

I think you'll find as much variety in flower color genetics as in any other area of genetics. So, if you have parents of two different colors, you may have a strict dominant-recessive relationship, where 3/4 of offsprings' flowers are the dominant color and 1/4 are the recessive, or some other ratio, if it's a multi-gene interaction, or perhaps you'll get a blended or variegated variety where there isn't a dominance-recessive relationship. StuRat (talk) 21:22, 7 July 2011 (UTC)[reply]

Question about the sun

It's acknowledged that a human couldn't survive long on the Sun not to mention Earth itself. My question is since Jupiter and Saturn are bigger than any of the other planets in our solar system, if the Sun were to consume Jupiter and Saturn, how long would it take for the sun's heat to vanquish the two planets? My question applies to both if the sun were to swallow them at the same time or subsequently, by the way. SwisterTwister talk 23:08, 7 July 2011 (UTC)[reply]

As our Roche limit article explains (although not very understandably), a planet such as Jupiter or Saturn could not even reach the Sun before being torn apart by tidal forces -- so they would be "vanquished" before they even got to the heat. Looie496 (talk) 23:18, 7 July 2011 (UTC)[reply]
I'll explain if the article is insufficient: basically, we know that the closer an object is to the Sun, the faster it completes one revolution around the Sun. This is due to the stronger pull of gravity the closer you get to the Sun. Therefore, if you think about a planet as a 3-d sphere instead of a point mass, you will realize that the point closest to the Sun on a planet will be pulled slightly harder than a point on the far side, leading to internal stresses within the planet. Now, for a faraway and small planet like Earth, the stress is not nearly enough to overcome the gravitational pull holding the Earth together. However, as you get closer and closer to a heavy body (like the Sun in this example), the gravitational force is increasing more and more rapidly, and the difference in forces between the near and far side of the planet get greater and greater. At some point, the difference in gravitational force will be too much for the planet to bear, and it will break apart into smaller pieces. The point at which this happens is called the Roche limit.
See, astrophysics doesn't have to be scary! -RunningOnBrains(talk) 23:26, 7 July 2011 (UTC)[reply]
The Roche limit varies as a function of density and as the lighter outer layers of the planets are removed the average density of the planet would increase eventually becoming high enough to stop the disintegration (That would only be true for the rocky core likely present at the center of those planets). Dauto (talk) 02:01, 8 July 2011 (UTC)[reply]
But suppose you used duct tape to secure all the surface of the planet before you sent it towards the Sun? Myles325a (talk) 06:38, 8 July 2011 (UTC)[reply]
Even if Jupiter and Saturn had a surface, duct tape would have a negligible effect compared to the gravitational forces involved. There are some things that even duct tape cannot fix.--Shantavira|feed me 07:28, 8 July 2011 (UTC)[reply]
What if the duct tape is made of Quarkium? (See a section farther up the page). As regards the concept of duct-taping Jupiter and Saturn, that's a funny mental picture. Where's Red Green when you need him? :) ←Baseball Bugs What's up, Doc? carrots08:29, 8 July 2011 (UTC)[reply]
Well, if you used quarkium duct tape, then the tape might not fail, but the glue still would. Googlemeister (talk) 15:46, 8 July 2011 (UTC)[reply]
Some simulations suggest that the Sun's final transition to planetary nebula will blow off the outer gaseous atmosphere of Jupiter and Saturn, although Saturn's rings may have collapsed by then. ~AH1 (discuss!) 15:47, 8 July 2011 (UTC)[reply]

July 8

Climate in Kuwait

I have a friend living in Kuwait City, which is currently experiencing temperatures bordering on 50 degrees celsius. Where does this place rank globally in terms of summertime maximum and average temperatures? I've never heard of a place this hot. Thanks. 49.185.136.107 (talk) 03:04, 8 July 2011 (UTC)[reply]

It ranks pretty high, but there are many spots in that region that frequently reach similar temperatures in midsummer. The average temperatures are actually a lot lower, because the dryness and lack of vegetation produce a lot of cooling during the night. For what it's worth, the hottest place on Earth, in terms of average temperature, is said to be the Afar Depression in Ethiopa -- not all that far away from Kuwait. Looie496 (talk) 03:29, 8 July 2011 (UTC)[reply]
See Climate of Kuwait. The hottest recorded temperature in Kuwait was 52.6°C in Abdaly, on June 15, 2010[22]. ~AH1 (discuss!) 15:44, 8 July 2011 (UTC)[reply]

Nuclear reticulum

Is nuclear reticulum same as Nuclear lamina? --111Engo (talk) 04:07, 8 July 2011 (UTC)[reply]

No, the nuclear lamina is the meshwork of proteins adjacent the phospholipd nuclear membrane. The nuclear reticulum is a term used to refer to a network like pattern seen in the nucleus. Depending on the context it may be the
  • the pattern of chromatin when it is stained
  • the distribution of ribonucleoproteins seen by immunostaining .
  • the tubules and vesicles formed by the nuclear membrane protruding into the centre of the nucleus.(think of this as an endoplasmic reticulum) in the nucleus.

(maybe the nuclear architecture article needs updating?)Staticd (talk) 07:57, 8 July 2011 (UTC)[reply]

Hello, a question about the article "Wilson's disease"

it says (in the part; "nutrition"); "In general, a diet low in copper-containing foods is recommended, with the avoidance of mushrooms, nuts, chocolate, dried fruit, liver, and shellfish".[1]

my Q;

Nuts, what kind of nuts?, and also, organic for example also contain copper?.

and mushrooms, what kind?.

BTW, is it true that some plants organically contain copper? — Preceding unsigned comment added by 79.182.28.217 (talk) 07:31, 8 July 2011 (UTC)[reply]

All plants contain copper as it is part of many essential enzymes. However the levels vary depending on the species, the part and how it was grown. Regarding what foods to avoid, that would be medical advice, you would have to ask a doctor. It's taboo to offer it here. :). Staticd (talk) 08:01, 8 July 2011 (UTC)[reply]
It's a little bit late telling the OP we can't help with dietary recommendations, when those he listed above came straight from Wikipedia anyway. Even the title of this section says the question is about the Wilson's disease article just as much as the disease itself. And that list of foods to avoid IS questionable, including, as it does, nuts. That's such a catch-all category of foods in English. It includes things as diverse as peanuts, pistachios, walnuts and macadamias. To assign a common characteristic to them all is not very scientific. Given what the OP has already garnered from Wikipedia, I think anyone who can is almost obliged to help clarify further. HiLo48 (talk) 09:07, 8 July 2011 (UTC)[reply]
The book Wilson's disease: a clinician's guide to recognition, diagnosis, and management, (from 2001 though) suggests that diet isn't important apart from liver and shellfish which can be significantly high in copper. See page 73. Sean.hoyland - talk 10:25, 8 July 2011 (UTC)[reply]

Electric Power

IN DIRECT CURRENT,flow of electron is continues i.e.,from negative terminal to positive terminal.but in ALTERNATING CURRENT, electron moves forward and again backward but does not move from negative terminal to the positive terminal so that only current wave travel like when we throw a stone in a river then wave only travels not the water particle. is it true or not. PLEASE GIVE ME BRIEF DETAIL ABOUT ELECTRON FLOW IN A.Cvsnkumar (talk) 12:20, 8 July 2011 (UTC)[reply]

Have a look at Alternating current and Wave. Dolphin (t) 12:44, 8 July 2011 (UTC)[reply]
You seem to have the right idea. For both AC and DC, it is the movement of the charges ("current") that matters, not where they end up. In AC, electrons move into the terminals and out of them again. The electrons do work on the load both when they move in and when they move out. --Srleffler (talk) 17:31, 8 July 2011 (UTC)[reply]

Electric power2

I put a Electric tester in a socket's neutral,it shows zero voltage.we know that Alternating current is a sinusoidal.i.e.,for positive cycle it travels in one direction and for negative half cycle it travel in opposite direction.according to this,is phase and neutral reverses for every cycle in a socket or not? if not how electrons flow in a conductor for A.C. Also in d.c, please give me brief detail about electron flow in a.c according to my questionvsnkumar (talk) 12:23, 8 July 2011 (UTC)[reply]

Have a look at Alternating current. Dolphin (t) 12:46, 8 July 2011 (UTC)[reply]
The electrons flow into and out of both the live and neutral sockets, however this flow is driven by the live socket. The electric field in the live socket pushes electrons into the wire and pulls them out of it. The flow of electrons into and out of the wire pushes and pulls electrons along the whole length of the wire, through the connected device, and back up the wire to the neutral socket. This pushes electrons into the neutral socket and pulls them out of it, on alternating halves of the cycle.--Srleffler (talk) 17:35, 8 July 2011 (UTC)[reply]

Inertial mass, again

This is based on a question I asked earlier.

E=mc2 means that the potential energy between particles will manifest as mass. But if the potential energy function is defined up to a constant, then shouldn't the mass of this potential energy likewise be not uniquely determined? What would the actual mass be? — Preceding unsigned comment added by 74.15.136.219 (talk) 15:25, 8 July 2011 (UTC)[reply]

In relativity, unlike in classical mechanics, energy isn't defined up to a constant. (just like classically, temperature was only defined up to a constant (calcius, farenheit) but now we know better and have an absolute scale (kelvin) ). 213.49.89.68 (talk) 15:32, 8 July 2011 (UTC)[reply]

The flagpole experiment

Hi. Imagine about two dozen people gathered arms stretched out, holding hands in a circle centred on a flagpole. Simultaneously, all the participants run toward the centre of the flagpole, keeping their arms streched out. What would occur in this instance? Would the people start rotating in a circle? If this is the case, in what direction would the rotation most likely be psychologically? Would the same occur of all the people were running outwards or facing inwards and running backwards? What would be the average speed of the revolution? Thanks. ~AH1 (discuss!) 15:34, 8 July 2011 (UTC)[reply]

Well if they must keep their arms stretched out and if they are not allowed to change the angle of their arms, then the circle will somehow be broken. I don't see how motion towards the center of a circle will translate into a rotation in this scenario. Googlemeister (talk) 15:44, 8 July 2011 (UTC)[reply]
Same response: arms outstretched equals an impossibility as zeroing in on the flagpole progresses beyond a certain degree. But I don't think rotation of the group around the flagpole has to take place. Bus stop (talk) 15:47, 8 July 2011 (UTC)[reply]
I see people falling over but no general rotation. The circle ceases to be a circle as the stronger push aside the weaker. One person reaches the flagpole, though the center of the flagpole is out of reach. In the case of running away from the flagpole the pair of hands with the weakest hold break first. If every runner follows exactly a radial from the flagpole, all the handholds have to break. In any practical experiment some handholds will last longer than others as runners veer off course. Cuddlyable3 (talk) 17:29, 8 July 2011 (UTC)[reply]

Plant identification

While out chopping down burdocks from the roadside (they get in the dog's fur when on walks), I saw a bunch of these. Can anyone identify them? The pic was taken in Vermont and just a few minutes ago. Dismas|(talk) 16:06, 8 July 2011 (UTC)[reply]

I think it's wild grape. We had some in my back yard as a kid in Connecticut. -RunningOnBrains(talk) 17:16, 8 July 2011 (UTC)[reply]
I don't think it is a grape, wild or tame, because the leaf stalks appear to come from a central point which is not a characteristic of a grape which grows leaves along a rambling stem. It looks like it might be petasites, more commonly known as butterbur, specifically the white butterbur (petasites albus). The temperate geographical zone is right. Certain identification is always difficult until one can see the flower and/or the whole mature plant. Richard Avery (talk) 17:53, 8 July 2011 (UTC)[reply]
Let it be known that I am by no means a plant expert, so I default to Richard Avery's assessment. The leaves just looked very similar to what I remembered (and the website I saw). -RunningOnBrains(talk) 18:27, 8 July 2011 (UTC)[reply]
I don't remember ever seeing them bloom (they do a decent job of cutting down the weeds on the roadsides around here) but I think Richard has it. These don't have any tendrils like the grapes do. So, White Butterbur it is then! Thanks, Dismas|(talk) 21:40, 8 July 2011 (UTC)[reply]

is using a whiteboard and markers a health hazard?

Is using a whiteboard and whiteboard markers a health hazard? (cancerous, small particles into lungs, etc etc). Not asking for medical advice. 188.29.193.67 (talk) 16:19, 8 July 2011 (UTC)[reply]

I'm not sure about small particles. The pigment solvent you are asking about is mostly harmless. I hope it was tested on laboratory animals. Are white board users acting strange? Remind them not to concentrate or inhale the solvent. If the concentration is such as to cause a headache or euphoria, move to a well-ventilated area. This is not medical advice. 99.24.223.58 (talk) 18:24, 8 July 2011 (UTC)[reply]

Radio waves

Do radio waves have angular momentum? This is unclear. Cuddlyable3 (talk) 16:57, 8 July 2011 (UTC)[reply]

Yes. Not that I have any idea what that means: Photon_polarization#Angular_momentum_density_of_classical_electromagnetic_waves. -RunningOnBrains(talk) 17:12, 8 July 2011 (UTC)[reply]
Every photon carries a momentum equal to Planck's constant. So radio waves actually carry much more angular momentum than an equivalent mass-energy of visible light. Wnt (talk) 19:16, 8 July 2011 (UTC)[reply]

Current plan for NASA

What is the current (perhaps read yet-to-be-cancelled) plan for NASA that isn't cancelled? I know there used to be Constellation, but that is no longer. 20.137.18.50 (talk) 17:57, 8 July 2011 (UTC)[reply]

For human space flight, robots to clear the dangerous Low earth orbit#Space debris. Those can probably be similar to the robots which weld asteroids together for shielding from cosmic radiation on extended trips outside the LEO's Van Allen belts. The most cost effective James Webb Space Telescope replacement is a far side lunar cryostat cooled infrared VLBI array capable of detecting exoplanet ozone and the first two infrared spectral lines of hydrogen. 99.24.223.58 (talk) 18:11, 8 July 2011 (UTC)[reply]
The Space Launch System is the current project filling the role of the now-cancelled Ares rocket. Note that, like Ares, it is a Shuttle-Derived Launch Vehicle. Similarly, Constellation's Orion capsule has morphed into the Multi-Purpose Crew Vehicle program. — Lomn 18:20, 8 July 2011 (UTC)[reply]
Last I heard, the MPCV reentry testing was scheduled for 2013. Is that still on? 99.24.223.58 (talk) 18:26, 8 July 2011 (UTC)[reply]
Also, is NASA going to do a synthetic fuel demonstration? I'd love to see that. 99.24.223.58 (talk) 20:14, 8 July 2011 (UTC)[reply]

There's sometimes a very easy way to answer questions like the OP's. Just yesterday I went to nasa.gov, and on the very first screen was a button labelled What's next? Suggest you look there. HiLo48 (talk) 23:14, 8 July 2011 (UTC)[reply]

Another good question is what is NASA's track record for programs accomplishing the goals they set out to accomplish before the projects get canceled. 76.27.175.80 (talk) 00:50, 9 July 2011 (UTC)[reply]
That depends on what you think the goals are, and I suspect the goals might be far more advanced than what most people think they are. 99.24.223.58 (talk) 01:49, 9 July 2011 (UTC)[reply]
You mean they don't by habit state clear goals outright with time tables such as "we will put a man on the moon by the end of this decade" (though of course not always as big)?76.27.175.80 (talk) 02:08, 9 July 2011 (UTC)[reply]

Does special relativistic length contraction "hide" matter?

I get that if something is travelling at the speed of light (relative to you), it has zero length in the direction of travel. But does that mean that there could be complex, large objects that are "collapsed" out of sight because they are travelling too quickly for you to see? I guess I don't understand whether photons are the only things that travel at the speed of light, and how it is that we can "see" photons if they have zero length. johnpseudo 18:47, 8 July 2011 (UTC)[reply]

We don't see photons! We see other things, by way of the photons they emit or reflect. Photons themselves are invisible, because (barring extremely minor quibbles) they neither scatter nor emit photons. --Trovatore (talk) 22:57, 8 July 2011 (UTC)[reply]
Nonsense. Photons are all that we do see. Everything else is inferred. Photons that don't hit your retina are invisible, of course, but you wouldn't say "we don't taste salt" just because most salt never enters your mouth. -- BenRG (talk) 00:44, 9 July 2011 (UTC)[reply]
The claim that "photons are all we do see" is incorrect. Right now I see, for example, my keyboard. My keyboard is not made of photons.
This is a linguistic point, of course, not a physical one. Nevertheless I think I have a very good argument here. When a photon hits your retina, you do not see the photon (if you did, what shape was it?) You see what caused the photon to enter your eye. --Trovatore (talk) 00:51, 9 July 2011 (UTC)[reply]
Well, the classical approximation is either that a photon is a point particle with zero length, width, and breadth; or else that it is a wave that diffuses out for some distance in all directions. Quantum mechanics combines this in particle-wave duality. But the point is, the probability distribution or waveform of the photon isn't compressed down to nothing, because there's an uncertainty in the time when it was emitted, not just a physical length. Though the difference between these might be interesting to consider... for example, I suppose a particle of slow light, crawling through a medium at some few km/h, must be much more compressed from front to back than it is in free space. Odd... Wnt (talk) 19:15, 8 July 2011 (UTC)[reply]
Also, to answer your other question, nothing with mass can ever reach the speed of light, where it would theoretically have zero length in a "stationary" rest frame, since it would take an infinite amount of energy to do so. If you managed to somehow accelerate a macroscopic object to near the speed of light (impossible with today's technology), it would appear massively distorted (shrunk) in the direction of travel, but all scales would be affected; even down to the atoms themselves. No matter is "hidden". You might be interested in the ladder paradox.-RunningOnBrains(talk) 19:37, 8 July 2011 (UTC)[reply]
Don't confuse the Lorentz contraction with the actual visual appearance of the object. That (to the extent that you'd have time to see it) is different, because light reaching you from different parts of the object was reflected at different times (in your inertial coordinate system).
An approximation is to say that the object appears rotated, rather than shrunk. I am not sure how well "rotated" really captures the exact distortions involved. We should probably have an article on this, if we don't. Here is the classic paper on the subject. --Trovatore (talk) 22:02, 8 July 2011 (UTC)[reply]
Although that is the classic paper, in the sense that it's the one that everyone cites, I think it does a poor job of explaining the situation. It uses two objects at the same location but moving at different speeds, and a single stationary observer. It's much easier if you use a single stationary object and two observers. Then it's easy to see that the two observers will see the same view of the object, because their eyes receive the same light. If the object is a cube, then there is a region of space which is only reached by light from one face, and if they are in that region of space then they will both see only that face. They won't both see it as square, because of the aberration of light. But relativistic aberration is conformal (preserves angles), so they will both see 90° angles at the four corners of the shape they do see. Conformal transformations also preserve circles, so if you draw a circle on the face that touches the four edges, both observers will see it as a circle touching all four edges. This gives you an idea of the limits of the distortion introduced by aberration. There's no way it can "length contract" the square, for example, because you can't draw a circle inside a narrow rectangle that touches all four edges. In fact what you get is a trapezoid-like shape, with curved edges that meet at 90° angles. All of this is true even if the object is close by, contrary to Weisskopf's analysis which only works when it's far away.
So why does Weisskopf say that the object is rotated? Because he's treating the two objects as coincident when the emit the light, while I'm treating the observers as coincident when they receive the light, and these aren't the same thing. If you split Weisskopf's observer into two separate observers in two separate experiments, each with one object, and boost so that both objects are at rest in the same place, the observers aren't in the same place when they receive the light, so they really are seeing the object from different angles (in addition to the distortion introduced by aberration).
The behavior in the speed-of-light limit depends on how you take the limit. If you do it Weisskopf-style, with the object suddenly accelerating to high speed, you see a highly redshifted and slightly distorted square of pretty much the same size as before. It may be redshifted out of detectability, though. It doesn't shrink away, but does fade away. If you take the limit my style, with the observer accelerating to high speed, the object shrinks to a point (the headlight effect), blueshifts, and actually gets brighter overall. It shrinks away, but doesn't fade away. So this question has no easy answer; it depends on how you set it up. -- BenRG (talk) 00:44, 9 July 2011 (UTC)[reply]

Picture quality in digital cameras

Which factors are the most important to choose? I know that the MBs are only secondary, and that the lens is probably the most important, but what else should I choose? Wikiweek (talk) 21:53, 8 July 2011 (UTC)[reply]

That depends almost entirely on what you intend to use it for. --Tango (talk) 22:43, 8 July 2011 (UTC)[reply]
Are you asking, perchance, that you should obtain a camera which is not also a cell phone? I can not say. 99.24.223.58 (talk) 23:46, 8 July 2011 (UTC)[reply]
I know that cell-phone cameras are horrible. I intend to use the camera for making pictures, I don't know what are other options. Wikiweek (talk) 00:24, 9 July 2011 (UTC)[reply]
For sensors of a given generation, the single biggest impact on image quality is pixel density: a big sensor with a small megapixel count will produce a better image than a small sensor with a high count. Larger sensors in general do better than smaller ones because of this. --Carnildo (talk) 00:32, 9 July 2011 (UTC)[reply]

What looks like peas? We may soon know

I asked the question here and here and was referred here in the second section. The conclusion was that it was Crape myrtle, and what I believe to be Crape myrtles are now blooming. The trees near my house are not blooming yet and I won't be going to Myrtle Beach, South Carolina until the blooms are gone and what looks like peas are back. I'll update this as the trees bloom.Vchimpanzee · talk · contributions · 21:54, 8 July 2011 (UTC)[reply]

Peas are pulses which would be fruits if they had not been defined to be vegetables. Long story. 99.24.223.58 (talk) 22:45, 8 July 2011 (UTC)[reply]
What's your point, exactly? --Mr.98 (talk) 23:28, 8 July 2011 (UTC)[reply]
Well, that is a difficult question having to do with the fact that vegetables and fruits were named before biology was understood. 99.24.223.58 (talk) 00:04, 9 July 2011 (UTC)[reply]

July 9

  1. ^ PMID 8210311
  2. ^ PMID 18046752
  3. ^ Elsimar M. Coutinho and Sheldon J. Segal (1999). Is menstruation obsolete?. {{cite book}}: Unknown parameter |Publisher= ignored (|publisher= suggested) (help)
  4. ^ Paul Bischof and Marie Cohen. "Course 4:Implantation" (PDF). European Society of Human Reproduction and Embryology.